SlideShare a Scribd company logo
1 of 34
Krok 1 Medicine
4.0 Нормальна фізіологія
1
Patient with hypersecretion of the gastric juices was recomended to exclude from the
diet
concentrated bouillons and vegetable decoctions because of their stimulation of gastric
secretion. What is dominating mechanism of stimulation of secretion in this case?
A Stimulation of gastrin production by G-cells *
B Irritation of taste receptors
C Irritation of mechanoreceptors of the oral cavity
D Irritation of mechanoreceptors of the stomach
E Stimulation of excretion of secretin in the duodenum
2
Person felt thirsty after staying in heat for a long time. Signals of what receptors
caused it first
of all?
A Osmoreceptors of hypothalamus *
B Sodium receptors of hypothalamus
C Osmoreceptorsof the liver
D Glucoreceptors of hypothalamus
E Baroreceptors of aortic arch
3
While emotional excitement the heart rate in a 30-year-old person run up to 112 Bpm.
What part
of the conducting system of the heart caused it?
A Synoatrial node *
B Purkinje's fibers
C His bundle branches
D Intraventricular node
E His bundle
4
A 38-year-old woman was admitted to the admission-diagnostic department with
uterine
bleeding. What are the most likely changes of blood?
A Reduction of haematocrite rate *
B Increase of haematocrite rate
C Leukopenia
D Leucocytosis
E Polycythemia
5
Due to action of electric current on the exitable cell there appeared depolarization of it's
membrane. Movement of what ions through the membrane caused depolarisation?
A Na+
B НСО3
-
CСа2+
D Сl-
EК+
6
On experiment on the dog the peripheral part of nervus vagus of the neck was irritated. What
changes of the heart function would be observed?
ADecreased contraction rate *
BIncreased contraction force
CIncreased atrioventricular conduction
D Increased contraction force and rate
EIncreased myocardial excitability
7
Power inputs of a boy increased from 500 to 2000 kJ pro hour. What can be the cause of it?
APhysical exercise *
BRaise of outer temperatute
CMental activity
D Food intake
ETransition from sleep to wakefulness
8
In a young man during exercise, the minute oxygen uptake and carbon dioxide emission
equalled to 1000 ml. What substrates are oxidized in the cells of his body?
ACarbohydrates *
BProteins
CFats
D Carbohydrates and fats
ECarbohydrates and proteins
9
In the experiment on the animal the part of the cerebral cortex hemispheres was removed. It
caused elimination of previously formed conditioned reflex to the light irritation. What part ot the
cortex was removed?
AOccipital cortex *
BPrecentral convolution
CPostcentral convolution
D Limbic cortex
ETemporal lobe
10
Inhibition of alpha-motoneuron of the extensor muscles was noticed after stimulation of
alpha-motoneuron of the flexor muscles during the experiment on the spinal column. What type
of inhibition can this process cause?
AReciprocal *
BPresynaptic
CDepolarizational
D Recurrent
ELateral
11
Respiratory coefficient was studied in the patient who strictly kept certain diet for 10 days. It
was determined that it is 1. What diet does the patient follow?
AWith domination of carbohydrates *
BWith domination of proteins and fat
CWith domination of fat and carbohydrates
D Mixed
EWith domination of proteins and carbohydrates
12
A sportsman spontaneously held breath for 40 seconds, which resulted in an increase in heart
rate and systemic arterial pressure. Changes of these indicators are due to activation of the
following regulatory mechanisms:
AUnconditioned sympathetic reflexes *
BUnconditioned parasympathetic reflexes
CConditioned sympathetic reflexes
D Conditioned parasympathetic reflexes
E-
13
Punctata hemorrhage was found out in the patient after application of a tourniquet. With
disfunction of what blood cells is it connected?
APlatelets *
BEosinophiles
CMonocytes
D Lymphocytes
ENeutrophiles
14
Students who are taking examinations often have dry mouth. The mechanism that causes this
state is the realization of the following reflexes:
AConditioned sympathetic *
BUnconditioned parasympathetic
CConditioned parasympathetic
D Unconditioned sympathetic
EUnconditioned peripheral
15
Middle part of cochlear of internal ear was destroyed in animal while experiment. It will cause
abnormalities of the sound perception of the following frequencies:
AMiddle *
BLow
CHigh
D High and low
ENo abnormalities
16
The temperature of the ambient environment is 38oC and relative air humidity is 50%. What
ways of heat emission provide maintaining a constant temperature of the human body?
AEvaporation *
BRadiation
CHeat conduction
D Convection
EConvection and conduction
17
The minute blood volume in a patient with transplanted heart has increased as a result of
physical activity. What regulative mechanism is responsible for these changes?
ACatecholamines *
BSympathetic unconditioned reflexes
CParasympathetic unconditioned reflexes
D Sympathetic conditioned reflexes
EParasympathetic conditioned reflexes
18
Isolated muscle of a frog is rhythmically irritated with electric impulses. Every next impulse is in a
period of relaxation from the previus contraction. What contraction of the muscle appears?
AWaved tetanus *
BSingle
CAsynchronous
D Continuous(smooth) tetanus
ETonic
19
A 30 year old woman has subnormal concentration of enzymes in the pancreatic juice. This
might be caused by the hyposecretion of the following gastrointestinal hormone:
ACholecystokinin-pancreozymin *
BSomatostatin
CSecretin
D Gastro-inhibiting peptide
EVaso-intestinal peptide
20
A patient has a trauma of sternocleidomastoid muscle. This caused a decrease in value of the
following indicator of external respiration:
AInspiratory reserve volume *
BExpiratory reserve volume
CRespiratory capacity
D Residual volume
EFunctional residual lung capacity
21
A man has normal sensitivity of his finger skin, however he doesn’t sense his wedding ring
around the finger. What process induced by wearing of the ring has caused this phenomenon?
AReceptor adaptation *
BDevelopment of the fibrous tissue
CAbnormality of the epidermis structure
D Impaired circulation
EAbnormality of the receptor structure
22
An aged man had raise of arterial pressure under a stress. It was caused by activation of:
ASympathoadrenal system *
BParasympathetic nucleus of vagus
CFunctions of thyroid gland
D Functions of adrenal cortex
EHypophysis function
23
A month after surgical constriction of rabbit's renal artery the considerable increase of
systematic arterial pressure was observed. What of the following regulation mechanisms
caused the animal's pressure change?
AAngiotensin-II *
BVasopressin
CAdrenaline
D Noradrenaline
ESerotonin
24
A child has abnormal formation of tooth enamel and dentin as a result of low concentration of
calcium ions in blood. Such abnormalities might be caused by deficiency of the following
hormone:
AParathormone *
BThyrocalcitonin
CThyroxin
D Somatotropic hormone
ETriiodothyronine
25
A sportsman was examined after an intensive physical activity. The examination revealed
disorder of movement coordination but the force of muscle contractions remained the same. It
can be explained by retarded speed of excitement conduction through:
ACentral synapses *
BNeuromuscular synapses
CEfferent nerves
D Afferent nerves
EConduction tracts
26
After a long training session a sportsman has developed fatigue accompanied by abrupt
performance decrement. What link of the reflex arch was the fatigue initiated in?
ANerve centres *
BAfferent conductor
CReceptors
D Efferent conductor
EMuscles
27
ECG study showed that the $T$-waves were positive in the standard extremity leads, their
amplitude and duration were normal. The right conclusion would be that the following process
runs normally in the heart ventricles:
ARepolarization *
BDepolarization
CExcitement
D Contraction
ERelaxation
28
Blood minute volume of a 30 year old woman at rest is 5 l/m. What blood volume is pumped
through the pulmonary vessels per minute?
A5 l *
B3,75 l
C2,5 l
D 2,0 l
E1,5 l
29
As a result of long-term starvation the glomerular filtration of a man was accelerated by 20%.
The most probable cause of filtration changes under such conditions is:
AFall of oncotic pressure of blood plasma *
BRise of systemic arterial pressure
CIncreased permeability of renal filter
D Growth of filtration coefficient
EIncrease of renal plasma flow
30
In course of an experiment a skeletal muscle is being stimulated by a series of electric impulses.
What type of muscle contraction will arise, if every subsequent impulse comes in the period of
shortening of the previous single muscle contraction?
AHolotetanus *
BPartial tetanus
CAsynchronous tetanus
D A series of single contractions
EMuscle contracture
31
A patient under test was subjected to a moderate physical stress. His minute blood volume
amounted 10 l/min. What blood volume was pumped through his lung vessels every minute?
A10 l/min *
B5 l/min
C4 l/min
D 6 l/min
E7 l/min
32
A patient presents with the following motor activity disturbances: tremor, ataxia and asynergia
movements, dysarthria. The disturbances are most likely to be localized in:
ACerebellum *
BBasal ganglions
CLimbic system
D Brainstem
EMedulla oblongata
33
A man has a considerable decrease in diuresis as a result of 1,5 l blood loss. The primary
cause of such diuresis disorder is the hypersecretion of the following hormone:
AVasopressin *
BCorticotropin
CNatriuretic
D Cortisol
EParathormone
34
An animal experiment is aimed at studying the cardiac cycle. All the heart valves are closed.
What phase of the cycle is characterized by this status?
AIsometric contraction *
BAsynchronous contraction
CProtodiastolic period
D Rapid filling
EReduced filling
35
While shifting the gaze to the closely situated object the refracting power of eye's optical
mediums will increase by 10 diopters. It results from changing of such eye structure:
ALens *
BCornea
CVitreous body
D Liquid of the anterior chamber of eye
EMuscle that dilatates pupil
36
Spasm of smooth muscle of bronchi developed in the patient. Usage of activators of what
membrane cytoreceptors is fisiologically valid to decrease attack?
Aβ-adrenoreceptors *
Bα-аdrenoreceptors
Cα- та β-аdrenoreceptors
D Н-cholinoreceptors
EМ-cholinoreceptors
37
On examination of the person it was revealed that minute volume of heart is 3500mL, systolic
volume is 50 mL. What is the frequency of cardiac contraction?
A70 bpm *
B60 bpm
C50 bpm
D 80 bpm
E90 bpm
38
Due to activation of ion channels of external membrane of excitable cell it's rest potential has
significantly increased. What channels were activated?
APotassium channels *
BNatrium channels
CFast calcium channels
D Slow calcium channels
ENatrium and calcium channels
39
The ventral roots of 5 frontal segment of spinal cord were cut during experiment in the animal.
What changes will take place in the innervation region?
ALoss of movements *
BLoss of touch sensitivity
CLoss of temperature sensitivity
D Loss of proprioceptive sensitivity
EHypersensitivity
40
Glomerular filtration rate (GFR) increased for 20% due to prolonged starvation of the person.
The most likely cause of filtration changes under this conditions is:
ADecrease of oncotic pressure of blood plasma *
BIncrease of systemic blood pressure
CIncrease of penetration of the renal filter
D Increase of filtration coefficient
EIncrease of renal plasma stream
41
A patient has a transverse disruption of spinal cord below the IV thoracic segment. What
changes of respiration will it cause?
ARespiration will stay unchanged *
BRespiration will stop
CRespiration will become less frequent
D Respiration will become deeper
ERespiration will become more frequent
42
A lightly dressed man is standing in a room, air temperature is +$14^0C$, windows and doors
are closed. In what way does he emit heat the most actively?
AHeat radiation *
BHeat conduction
CConvection
D Evaporation
EPerspiration
43
ECG of a patient with hyperfunction of thyroid gland showed heart hurry. It is indicated by
depression of the following ECG element:
AR-R interval *
BP-Q segment
CP-Q interval
D P-T interval
EQRS complex
44
A peripheral segment of vagus nerve on a dog's neck was being stimulated in course of an
experiment. The following changes of cardiac activity could be meanwhile observed:
AHeart rate fall *
BHeart hurry
CEnhancement of atrioventricular conduction
D Heart rate and heart force amplification
EIncreased excitability of myocardium
45
ECG of a patient shows prolongation of T-wave. This is caused by deceleration in ventricles of:
ARepolarization *
BDepolarization and repolarization
CDepolarization
D Contraction
ERelaxation
46
In a healthy adult speed of the excitement conduction through the atrioventricular node is
0,02-0,05 m/sec. Atrioventricular delay enables:
ASequence of atrial and ventricular contractions *
BSimultaneity of both atria contractions
CSimultaneity of both ventricles contractions
D Sufficient force of atrial contractions
ESufficient force of ventricular contractions
47
A 2 y.o. child has convulsions as a result of lowered concentration of calcium ions in blood
plasma. It is caused by reduced function of:
AParathyroid glands *
BHypophysis
CAdrenal cortex
D Pineal gland
EThymus
48
During preparation of a patient to a heart surgery it was necessary to measure pressure in
heart chambers. In one of them pressure varied from 0 mm Hg up to 120 mm Hg within one
cardiac cycle. What heart chamber is it?
ALeft ventricle *
BRight ventricle
CRight atrium
D Left atrium
E-
49
Heart rate of a man permanently equals 40 beats pro minute. What is the pacemaker?
AAtriventricular node *
BSinoatrial node
CHis' bundle
D His' bundle branches
EPurkinje's fibers
50
Stimulation of an excitable cell by the electric current has led to the depolarization of its
membrane. The depolarization has been caused mainly by the following ions penetrating into the
cell through its membrane:
ANa+
BHCO3-
CCa2+
D Cl-
EK+
51
Parents of a 10 y.o. boy consulted a doctor about extension of hair-covering, growth of beard
and moustache, low voice. Intensified secretion of which hormone must be assumed?
AOf testosterone *
BOf somatotropin
COf oestrogen
D Of progesterone
EOf cortisol
52
Lung ventilation in a person is increased as a result of physical activity. Which of the following
indices of the external respiration is much higher than in a state of rest?
ARespiratory volume *
BVital capacity of lungs
CInspiratory reserve volume
D Expiratory reserve volume
ETotal lung capacity
53
A man took a quiet expiration. Name an air volume that is meanwhile contained in his lungs:
AFunctional residual capacity *
BResidual volume
CExpiratory reserve volume
D Respiratory volume
EVital lung capacity
54
Examination of an isolated cardiomyocyte revealed that it didn't generate excitation impulses
automatically. This cardiomyocyte was obtained from:
AVentricles *
BSinoatrial node
CAtrioventricular node
D His' bundle
EPurkinje's fibers
55
Examination of a man established that cardiac output equaled 3500 ml, systolic output - 50 ml.
What is the man's heart rate pro minute?
A70 *
B60
C50
D 80
E90
56
As a result of continuous starvation the glomerular filtration rate has increased by 20%. The
most probable cause of the glomerular filtration alteration under the mentioned conditions is:
ADecrease in the oncotic pressure of blood plasma *
BIncrease in the systemic arterial pressure
CIncrease in the permeability of the renal filter
D Increase of the filtartion quotient
EIncrease of the renal blood flow
57
A man who went for a ride on a roundabout had amplification of heart rate, sweating and
nausea. What receptors stimulation is it primarily connected with?
AVestibular *
BProprioceptors
CTactors
D Auditory
EVisual
58
A man's intrapleural pressure is being measured. In what phase did the man hold his breath, if
his pressure is 7,5 cm Hg?
AQuiet inspiration *
BQuiet expiration
CForced inspiration
D Forced expiration
E-
59
Atria of an experimental animal were superdistended by blood that resulted in decreased
reabsorption of Na+ and water in renal tubules. This can be explained by the influence of the
following factor upon kidneys:
ANatriuretic hormone *
BAldosterone
CRenin
D Angiotensin
EVasopressin
60
A middle-aged man went to a foreign country because he had been offered a job there.
However he had been unemployed for quite a long time. What endocrine glands were exhausted
most of all in this man?
AAdrenal glands *
BParathyroid glands
CSeminal glands
D Substernal gland
EThyroid gland
61
A 42 year old patient complains of pain in the epigastral area, vomiting; vomit masses have the
colour of "coffee-grounds", the patient has also melena. Anamnesis records gastric ulcer. Blood
formula: erythrocytes - 2,8
*10
12/l, leukocytes – 8
*10
9/l, Hb- 90 g/l. What complication is it?
AHaemorrhage *
BPenetration
CPerforation
D Canceration
EPyloric stenosis
62
A human body cools in water much faster that in the air. What way of heat emission in water is
much more efficient?
AHeat conduction *
BConvection
CHeat radiation
D Sweat evaporation
E-
63
As a result of spinal-cord trauma a 33 y.o. man has a disturbed pain and temperature sensitivity
that is caused by damage of the following tract:
ASpinothalamic *
BMedial spinocortical
CPosterior spinocerebellar
D Lateral spinocortical
EAnterior spinocerebellar
64
After a surgery a 36-year-old woman was given an intravenous injection of concentrated
albumin solution. This has induced intensified water movement in the following direction:
AFrom the intercellular fluid to the capillaries *
BFrom the intercellular fluid to the cells
CFrom the cells to the intercellular fluid
D From the capillaries to the intercellular fluid
ENo changes of water movement will be observed
65
A clinic observes a 49 year old patient with significant prolongation of coagulation time,
gastrointestinal haemorrhages, subcutaneous hematomas. These symptoms might be explained
by the deficiency of the following vitamin:
AK *
BB
1
CB6
D H
EE
66
Examination of a patient revealed hyperkaliemia and hyponatremia. Low secretion of which
hormone may cause such changes?
AAldosteron *
BVasopressin
CCortisol
D Parathormone
ENatriuretic
67
Examination of a 43 y.o. anephric patient revealed anemia symptoms. What is the cause of
these symptoms?
AReduced synthesis of erythropoietins *
BEnhanced destruction of erythrocytes
CIron deficit
D Vitamin B12 deficit
EFolic acid deficit
68
While determining power inputs of a patient’s organism it was established that the respiratory
coefficient equaled 1,0. This means that in the cells of the patient the following substances are
mainly oxidized:
ACarbohydrates *
BProteins
CFats
D Proteins and carbohydrates
ECarbohydrates and fats
69
A patient has a disturbed absorbtion of fat hydrolysates. It might have been caused by a deficit
in the small intestine cavity:
AOf bile acids *
BOf bile pigments
COf lipolytic enzymes
D Of sodium ions
EOf liposoluble vitamins
70
Inhabitants of territories with cold climate have high content of an adaptive thermoregulatory
hormone. What hormone is meant?
AThyroxin *
BInsulin
CGlucagon
D Somatotropin
ECortisol
71
A concentrated solution of sodium chloride was intravenously injected to an animal. This caused
decreased reabsorption of sodium ions in the renal tubules. It is the result of the following
changes of hormonal secretion:
AAldosterone reduction *
BAldosterone increase
CVasopressin reduction
D Vasopressin increase
EReduction of atrial natriuretic factor
72
People adapted to high external temperatures have such pecularity: profuse sweating isn't
accompanied by loss of large volumes of sodium chloride. This is caused by the effect of the
following hormone upon the perspiratory glands:
AAldosterone *
BVasopressin
CCortisol
D Tgyroxin
ENatriuretic
73
The processes of heat transfer in a naked person at room temperature have been studied. It
was revealed that under these conditions the greatest amount of heat is transferred by:
AHeat radiation *
BHeat conduction
CConvection
D Evaporation
E-
74
During an experiment the dorsal roots of the spinal cord of an animal have been cut. What
changes will be observed in the innervation zone?
ASensitivity loss *
BLoss of motor functions
CDecrease in muscle tone
D Increase in muscle tone
ESensitivity loss and loss of motor functions
75
As a result of destruction of certain brainstem structures an animal has lost its orientative
reflexes in response to strong photic stimuli. What structures were destroyed?
AAnterior tubercles of quadrigeminal plate *
BPosterior tubercles of quadrigeminal plate
CRed nuclei
D Vestibular nuclei
EBlack substance
76
As a result of damage to certain structures of brainstem an animal lost orientation reflexes.
What structures were damaged?
AQuadritubercular bodies *
BMedial nuclei of reticular formation
CRed nuclei
D Vestibular nuclei
EBlack substance
77
Urine analysis has shown high levels of protein and erythrocytes in urine. This can be caused by
the following:
ARenal filter permeability *
BEffective filter pressure
CHydrostatic blood pressure in glomerular capillaries
D Hydrostatic primary urine pressure in capsule
EOncotic pressure of blood plasma
78
Osmotic pressure of a man's blood plasma is 350 mosmole/l (standard pressure is 300
mosmole/l). First of all it will result in high secretion of the following hormone:
AVasopressin *
BAldosteron
CCortisol
D Adrenocorticotropin
ENatriuretic
79
After a craniocerebral injury a patient is unable to recognize objects by touch. What part of brain
has been damaged?
APostcentral gyrus *
BOccipital lobe
CTemporal lobe
D Precentral gyrus
ECerebellum
80
A hypertensive glucose solution was introduced to a patient. It will intensify water movement:
AFrom the cells to the intercellular liquid *
BFrom the intercellular liquid to the capillaries
CFrom the intercellular liquid to the cells
D From the capillaries to the intercellular liquid
EThere will be no changes of water movement
81
To prevent long-term effects of 4-day malaria a 42-year-old patient was prescribed primaquine.
On the 3-rd day from the begin of treatment there appeared stomach and heart pains,
dyspepsia, general cyanosis, hemoglobinuria. What caused side effects of the preparation?
AGenetic insufficiency of glucose 6-phosphate dehydrogenase *
BCumulation of the preparation
CDecreased activity of microsomal liver enzymes
D Delayed urinary excretion of the preparation
EDrug potentiation by other preparations
82
According to audiometry data a patient has a disturbed perception of medium-frequency
sounds. It might have been caused by a damage of:
AMiddle part of helix *
BCochlear nuclei
CSpiral ganglion
D Quadritubercular structure
ELateral geniculate bodies
83
A 17-year-old boy fell seriously ill, body temperature rose up to 38,5
oС, there is cough,
rhinitis, lacrimation, nasal discharges. What kind of inflammation is it?
ACatarrhal inflammation *
BSerous inflammation
CFibrinous inflammation
D Suppurative inflammation
EHemorrhagic inflammation
84
A patient with disturbed cerebral circulation has problems with deglutition. What part of brain was
damaged?
ABrainstem *
BCervical part of spinal cord
CForebrain
D Interbrain
EMidbrain
85
A patient who has been treated with diazepam on account of neurosis complains of toothache.
Doctor administered him an analgetic, but its dose was lower than average therapeutic dose.
What phenomenon did the doctor take into account while prescribing the patient an underdose?
APotentiation *
BSummation
CCumulation
D Drug dependence
ETolerance
86
Long-term starvation cure of a patient resulted in diminished ratio of albumines and globulines in
plasma. What of the following will be result of these changes?
AIncrease of ESR *
BDecrease of ESR
CIncrease of hematocrit
D Decrease of hematocrit
EHypercoagulation
87
A patient has a decreased vasopressin synthesis that causes polyuria and as a result of it
evident organism dehydratation. What is the mechanism of polyuria development?
AReduced tubular reabsorption of water *
BReduced tubular reabsorption of Na ions
CReduced tubular reabsorption of protein
D Reduced glucose reabsorption
EAcceleration of glomerular filtration
88
A 35 year old man consulted a dentist about reduced density of dental tissue, high fragility of
teeth during eating solid food. This patient suffers the most probably from the deficiency of the
following mineral element:
ACalcium *
BPotassium
CSodium
D Magnesium
EIron
89
A patient is 44 years old. Laboratory examination of his blood revealed that content of proteins
in plasma was 40 g/l. What influence will be exerted on the transcapillary water exchange?
AFiltration will be increased, reabsorption - decreased *
BBoth filtration and reabsorption will be increased
CBoth filtration and reabsorption will be decreased
D Filtration will be decreased, reabsorption - increased
EExchange will stay unchanged
90
A patient has a critical impairment of protein, fat and hydrocarbon digestion. Most likely it has
been caused by low secretion of the following digestive juice:
APancreatic juice *
BSaliva
CGastric juice
D Bile
EIntestinal juice
91
After destruction of CNS structures an animal lost orientative reflexes. What structure was
destroyed?
AQuadrigeminal plate *
BRed nucleus
CLateral vestibular nuclei
D Black substance
EMedial reticular nuclei
92
An isolated cell of human heart automatically generates excitation impulses with frequency 60
times pro minute. What structure does this cell belong to?
ASinoatrial node *
BAtrium
CVentricle
D Atrioventricular node
EHis' bundle
93
A patient has a traumatic injury of sternocleidomastoid muscle. This has resulted in a decrease
in the following value:
AInspiratory reserve volume *
BExpiratory reserve volume
CRespiratory volume
D Residual volume
EFunctional residual lung capacity
94
Examination of a patient revealed a strong, balanced, inert type of higher nervous activity
according to Pavlov. What temperament type does the patient have (according to Hippocrates
classification)?
APhlegmatic *
BSanguine
CCholeric
D Melancholic
E-
95
The receptors under study provide transfer of information to the cortex without thalamic
involvement. Specify these receptors:
AOlfactory *
BTactile
CGustatory
D Visual
EAuditory
96
Examination of a patient revealed overgrowth of facial bones and soft tissues, tongue
enlargement, wide interdental spaces in the enlarged dental arch. What changes of the
hormonal secretion are the most likely?
AHypersecretion of the somatotropic hormone *
BHyposecretion of the somatotropic hormone
CHypersecretion of insulin
D Hyposecretion of thyroxin
EHyposecretion of insulin
97
A patient has a haemorrhage into the posterior central gyrus. What type of sensitivity on the
opposite side will be disturbed?
ASkin and proprioceptive *
BVisual
CAuditory
D Olfactory
EAuditory and visual
98
A 32-year-old patient consulted a doctor about the absence of lactation after parturition. Such
disorder might be explained by the deficit of the following hormone:
AProlactin *
BSomatotropin
CVasopressin
D Thyrocalcitonin
EGlucagon
99
During influenza epidemic 40% of pupils who didn't go in for sports were affected by the
disease, and among the pupils who regularly did physical exercises this index was only 20%.
What adaptative mechanisms determined such a low sickness rate of pupils participating in the
sports?
ACross adaptation *
BSpecific adaptation
CPhysiological adaptation
D Biochemical adaptation
EGenetic adaptation
100
A 60 year old patient was found to have a dysfunction of main digestive enzyme of saliva. This
causes the disturbance of primary hydrolysis of:
ACarbohydrates *
BFats
CProteins
D Cellulose
ELactose
101
A 49 year old woman spent a lot of time standing. As a result of it she got leg edema. What is
the most likely cause of the edema?
AIncrease in hydrostatic pressure of blood in veins *
BDecrease in hydrostatic pressure of blood in veins
CDecrease in hydrostatic pressure of blood in arteries
D Increase in oncotic pressure of blood plasma
EIncrease in systemic arterial pressure
102
A patient presented to a hospital with complaints about quick fatigability and significant muscle
weakness. Examination revealed an autoimmune disease that causes functional disorder of
receptors in the neuromuscular synapses. This will result in the disturbed activity of the following
mediator:
AAcetylcholine *
BNoradrenaline
CDopamine
D Serotonin
EGlycine
103
A 30-year-old woman was diagnosed with insufficiency of exocrinous function of pancreas.
Hydrolisis of what nutrients will be disturbed?
AProteins, fats, carbohydrates *
BProteins, fats
CProteins, carbohydrates
D Fats, carbohydrates
EProteins
104
During an animal experiment, surgical damage of certain brain structures has caused deep
prolonged sleep. What structure is most likely to cause such condition, if damaged?
AReticular formation *
BBasal ganglion
CRed nuclei
D Hippocampus
ECerebral cortex
105
Short-term physical activity resulted in reflex amplification of heart rate and raise of systemic
arterial pressure. What receptors activation was the main cause of pressor reflex realization?
AProprioreceptors of active muscles *
BVascular chemoreceptors
CVascular volume receptors
D Vascular baroceptors
EHypothalamus thermoreceptors
106
In course of an experiment a skeletal muscle is being stimulated by a series of electric impulses.
What type of muscle contraction will arise, if every subsequent impulse comes in the period of
relaxation of single muscle contraction?
APartial tetanus *
BHolotetanus
CA series of single contractions
D Muscle contructure
EAsynchronous tetanus
107
A patient takes cholagogues. What other process besides biliary excretion will be stimulated?
AIntestinal motility *
BGastric juice secretion
CPancreatic juice secretion
D Gastric motor activity
EWater absorption
108
Packed cell volume of a man was 40% before the trauma. What packed cell volume will be
observed 24 hours after blood loss of 750 ml?
A30%
B40%
C55%
D 45%
E50%
109
A patient staggers and walks astraddle. He has hypomyotonia of arm and leg muscles, staccato
speech. In what brain section is this affection localized?
ACerebellum *
BPutamen
CCaudate nucleus
D Motor cortex
ERed nucleus
110
A pregnant woman had her blood group identified. Reaction of erythrocyte agglutination with
standard serums of 0αβ (I), Bα (III) groups didn't proceed with standard
serum of Aβ (II) group. The blood group under examination is:
AАβ (II) *
B0αβ (I)
CВα (III)
D АВ (IV)
E-
111
Blood group of a 30 year old man was specified before an operation. His blood is Rh-positive.
Reaction of erythrocyte agglutination was absent with standard sera of 0αβ (I),
Аβ (II), Вα (III) groups. The blood under examination is of the following group:
A0αβ (I) *
BАβ (II)
CВα (III)
D АВ (IV)
E-
112
During an experiment the myotatic reflex has been studied in frogs. After extension in a skeletal
muscle its reflectory contraction was absent. The reason for it might be a dysfunction of the
following receptors:
AMuscle spindles *
BNociceptors
CArticular
D Golgi tendon organs
ETactile
113
Vagus branches that innervate heart are being stimulated in course of an experiment. As a
result of it the excitement conduction from atria to the ventricles was brought to a stop. It is
caused by electrophysical changes in the following structures:
AAtrioventricular node *
BHis' bundle
CSinoatrial node
D Ventricles
EAtria
114
If a man has an attack of bronchiospasm it is necessary to reduce the effect of vagus on
smooth muscles of bronchi. What membrane cytoreceptors should be blocked for this purpose?
AM-cholinoreceptors *
BN-cholinoreceptors
Cα-adrenoreceptors
D β-adrenoreceptors
Eα- and β-adrenoreceptors
115
When water affects mucous membrane of the inferior nasal meatuses, this causes "diver
reflex" that provokes:
AReflectory apnea *
BReflectrory dyspnea
CReflectory hyperpnea
D Cough
EBronchospasm
116
A man weighs 80 kg, after long physical activity his circulating blood volume is reduced down to
5,4 l, hematocrit makes up 50%, whole blood protein is 80 g/l. These blood characteristics are
determined first of all by:
AWater loss with sweat *
BIncreased number of erythrocytes
CIncreased protein concentration in plasm
D Increased circulating blood volume
EIncreased diuresis
117
A 16 year old boy after an illness has diminished function of protein synthesis in liver as a result
of vitamin $K$ deficiency. It will cause disturbance of:
ABlood coagulation *
BErythrocyte sedimentation rate
CAnticoagulant generation
D Erythropoietin secretion
EOsmotic blood pressure
118
In response to a change in body position from horizontal to vertical blood circulation system
develops reflectory pressor reaction. Which of the following is its compulsory component?
ASystemic constriction of the venous vessels *
BSystemic dilatation of the arterial resistive vessels
CDecrease in the circulating blood volume
D Increase in the heart rate
EWeakening of the pumbing ability of heart
119
Examination of a pregnant woman revealed twice as much concentration of fibrinogen in blood
plasm. What ESR can this woman have?
A40-50 mm/h *
B10-15 mm/h
C2-12 mm/h
D 5-10 mm/h
E0-5 mm/h
120
Introduction of a big dose of histamine to an experimental animal caused abrupt drop of arterial
pressure as a result of:
ADilatation of resistance vessels *
BConstriction of resistance vessels
CIncrease of heart rate
D Decrease of heart rate
EDecrease of heart rate and force
121
Systemic arterial pressure of an adult dropped from 120/70 to 90/50 mm Hg that led to
reflectory vasoconstriction. The vasoconstriction will be maximal in the following organ:
ABowels *
BHeart
CBrain
D Kidneys
EAdrenals
122
Microelectrode technique allowed to register a potential following "all-or-none" law and being
able of undecremental spreading. Specify this potential:
AAction potential *
BExcitatory postsynaptic potential
CRest potential
D Inhibitory postsynaptic potential
EReceptor potential
123
Vagus branches that innervate heart are being stimulated during an experiment. This caused
reduction of heart rate due to the intensification of the following process (through the cell
membrane of cardiac pacemaker):
APotassium ion yield *
BPotassium ion entry
CCalcium ion entry
D Calcium ion yield
ECalcium and potassium ion yield
124
Rest potential of a cell equals -80 mV. At what stage of action potential did the membrane
potential equal +30 mV?
AReverse polarization *
BAfter hyperpolarization
CAfter depolarization
D Depolarization
E-
125
A 35 year old man got an injury that caused complete disruption of spinal cord at the level of the
first cervical segment. What respiration changes will be observed?
AIt will come to a standstill *
BNo changes will be observed
CDiaphragmal respiration will be maintained, thoracic respiration will disappear
D Thoracic respiration will be maintained, diaphragmal respiration will disappear
EIt will become infrequent and deep
126
A doctor asked a patient to breath out fully after taking a normal breath. What muscles contract
during such exhalation?
AAbdominal muscles *
BExternal intercostal muscles
CDiaphragm
D Trapezius muscles
EPectoral muscles
127
A man was intoxicated with mushrooms. They contain muscarine that stimulates muscarinic
cholinoreceptors. What symptoms signalize intoxication with inedible mushrooms?
AMyotic pupils *
BMydriatic pupils
CBronchi dilatation
D Increased heart rate
ERise of arterial pressure
128
A man presents with increased heart rate, mydriatic pupils, dry mouth. This condition results
from the activation of the following system of function regulation:
ASympathetic *
BParasympathetic
CMetasympathetic
D Vago-insular
EHypothalamo-pituitary-adrenal
129
In course of an experiment a peripheral section of vagus of an expiremental animal is being
stimulated. What changes will be observed?
AHeart rate fall *
BHeart hurry
CPupil dilation
D Increase of respiration rate
EBronchi dilation
130
Voluntary breath-holding caused increase of respiration depth and frequency. The main factor
stimulating these changes of external respiration is:
AIncreased tension of CO
2 in blood *
BIncreased tension of O2 in blood
CDecreased tension of O2 in blood
D Decreased tension of CO2 in blood
EDecreased concentration of H
+ in blood
131
A patient has delayed conduction of excitement through the atrioventricular node. What changes
of ECG will be observed?
AProlongation of P-Q interval *
BProlongation of Q-S interval
CNegative T wave
D S-T-segment displacement
EProlongation of Q-T interval
132
Surface with an intact toad on it was inclined to the right. Tone of extensor muscles became
reflectory higher due to the activation of the following receptors:
AVestibuloreceptors of utricle and saccule *
BVestibuloreceptors of semicircular ducts
CMechanoreceptors of foot skin
D Photoreceptors of retina
EProprioreceptors
133
In course of an experiment a toad's right labyrinth was destroyed. It will cause amyotonia of the
following muscles:
ARight extensors *
BLeft flexors
CLeft extensors
D Right flexors
ERight and left extensors
134
An animal has an increased tonus of extensor muscles. This the result of intensified information
transmission to the motoneurons of the spinal cord through the following descending pathways:
AVestibulospinal *
BMedial corticospinal
CReticulospinal
D Rubrospinal
ELateral corticospinal
135
Workers of a hothouse farm work under conditions of unfavourable microclimate: air
temperature is +37oC, relative humidity is 90%, air speed is 0,2 m/s. The way of heat
emission under these conditions will be:
AEvaporation *
BHeat conduction
CConvection
D Radiation
EAll the ways
136
Lungs of a preterm infant have areas of atelectasis (pulmonary collapse). The main cause is:
ASurfactant deficiency *
BIncreased viscous resistance
CUnderdeveloped inspiration muscles
D Diminished force of surface tension of lungs
ESurfactant excess
137
Vagi of an experimental animal were cut on both sides. What respiration changes will be
observed?
AIt will become deep and infrequent *
BIt will become shallow and frequent
CIt will become deep and frequent
D It will become shallow and infrequent
ENo changes will be observed
138
A cardiac electric stimulator was implanted to a 75 year old man with heart rate of 40 bpm.
Thereafter the heart rate rose up to 70 bpm. The electric stimulator has undertaken the function
of the following heart part:
ASinoatrial node *
BAtrioventricular node
CHis' bundle branches
D His' bundle fibers
EPurkinje's fibers
139
A patient came to the hospital complaining about quick fatigability and apparent muscle
weakness. Examination revealed an autoimmune disease that causes disorder of functional
receptor condition in neuromuscular synapses. What transmitter will be blocked?
AAcetylcholine *
BNoradrenalin
CDopamine
D Serotonin
EGlycine
140
Which muscle contraction will be observed in the upper extremity during holding (not moving) a
load in a certain position?
AIsometric *
BIsotonic
CAuxotonic
D Concentric
EExcentric
141
Examination of a 35 year old patient revealed high acidity of gastric juice. What receptors
should be blocked in order to reduce it?
AHistamine *
Bα
1-adrenoreceptors
Cα2-adrenoreceptors
D β1-adrenoreceptors
Eβ2-adrenoreceptors
142
A young woman who entered a production department where it strongly smelt of paints and
varnishes had a bronchospasm. This reflex was caused by irritation of the following receptors:
AIrritant *
BJuxtaglomerular
CPleura receptors
D Central chemoreceptors
EPeripheral chemoreceptors
143
A 60-year-old patient presents with weakened peristaltic activity of the bowels. Which of the
following foodstuffs would stimulate peristalsis most of all?
ABrown bread *
BWhite bread
CMeat
D Lard
ETea
144
An isolated muscle fiber is under examination. It was established that the threshold of stimulation
force became significantly lower. What is the cause of this phenomenon?
AActivation of sodium channels of membrane *
BActivation of potassium channels of membrane
CInactivation of sodium channels of membrane
D Inactivation of potassium channels of membrane
EBlock of energy production in the cell
145
It was established that agglutination of the recipient’s blood erythrocytes had been caused by
the standard sera from the I and II groups. Serum from the III group as well as anti-Rh
serum hadn‘t provoke any agglutination. Which blood group and rhesus is allowed to be
transfused this recipient?
AB, α (III) Rh
-
BA, β (II) Rh-
C0, α,β, (I) Rh+
D AB (IV), Rh+
EAB (IV), Rh-
146
A patient consumed a lot of reach in proteins food that caused increase of rate of proteolytic
enzymes of pancreatic juice. It is also accompanied by increase of rate of the following enzyme:
ATripsin *
BPepsin
CEnterokinase
D Gastricsin
ERenin
147
In course of an experiment thalamocortical tracts of an animal were cut. What type of sensory
perception remained intact?
AOlfactory *
BAuditory
CExteroreceptive
D Visual
ENociceptive
148
Due to the use of poor-quality measles vaccine for preventive vaccination, a 1-year-old child
has developed an autoimmune renal injury. The urine was found to contain macromolecular
proteins. What process of urine formation has been disturbed?
AFiltration *
BReabsorption
CSecretion
D Reabsorption and secretion
ESecretion and filtration
149
In course of an experiment there has been an increase in the nerve conduction velocity. This
may be caused by an increase in the concentration of the following ions that are present in the
solution around the cell:
ANa+
BK+ and Cl-
CK+ and Na+
D Ca2+ and Cl-
ECa2+
150
A patient has increased thickness of alveolar-capillary membrane caused by a pathologic
process. The direct consequence will be reduction of the following value:
ADiffusing lung capacity *
BOxygen capacity of blood
CRespiratory minute volume
D Alveolar ventilation of lungs
EExpiratory reserve volume

More Related Content

What's hot

Krok 1 - 2010 Question Paper (General medicine)
Krok 1 - 2010 Question Paper (General medicine)Krok 1 - 2010 Question Paper (General medicine)
Krok 1 - 2010 Question Paper (General medicine)Eneutron
 
Krok 1 - 2008 Question Paper (General medicine)
Krok 1 - 2008 Question Paper (General medicine)Krok 1 - 2008 Question Paper (General medicine)
Krok 1 - 2008 Question Paper (General medicine)Eneutron
 
Krok 1 - 2015 (Biochemistry)
Krok 1 - 2015 (Biochemistry)Krok 1 - 2015 (Biochemistry)
Krok 1 - 2015 (Biochemistry)Eneutron
 
Krok 1 - 2014 (Biochemistry)
Krok 1 - 2014 (Biochemistry)Krok 1 - 2014 (Biochemistry)
Krok 1 - 2014 (Biochemistry)Eneutron
 
Krok 1 - 2014 (Anatomy)
Krok 1 -  2014 (Anatomy)Krok 1 -  2014 (Anatomy)
Krok 1 - 2014 (Anatomy)Eneutron
 
Krok 1 - 2015 (Physiology)
Krok 1 - 2015 (Physiology)Krok 1 - 2015 (Physiology)
Krok 1 - 2015 (Physiology)Eneutron
 
Krok 1 - 2006 Question Paper (General medicine)
Krok 1 - 2006 Question Paper (General medicine)Krok 1 - 2006 Question Paper (General medicine)
Krok 1 - 2006 Question Paper (General medicine)Eneutron
 
Krok 1 - 2012 Question Paper (General medicine)
Krok 1 - 2012 Question Paper (General medicine)Krok 1 - 2012 Question Paper (General medicine)
Krok 1 - 2012 Question Paper (General medicine)Eneutron
 
Krok 1 - 2015 (Anatomy)
Krok 1 - 2015 (Anatomy)Krok 1 - 2015 (Anatomy)
Krok 1 - 2015 (Anatomy)Eneutron
 
Krok 1 - 2011 Question Paper (General medicine)
Krok 1 - 2011 Question Paper (General medicine)Krok 1 - 2011 Question Paper (General medicine)
Krok 1 - 2011 Question Paper (General medicine)Eneutron
 
Krok 1 - 2009 Question Paper (General medicine)
Krok 1 - 2009 Question Paper (General medicine)Krok 1 - 2009 Question Paper (General medicine)
Krok 1 - 2009 Question Paper (General medicine)Eneutron
 
Krok 1 2014 - biochemistry
Krok 1   2014 - biochemistryKrok 1   2014 - biochemistry
Krok 1 2014 - biochemistryEneutron
 
Krok 1 - 2015 Base (General Medicine)
Krok 1 - 2015 Base (General Medicine)Krok 1 - 2015 Base (General Medicine)
Krok 1 - 2015 Base (General Medicine)E_neutron
 
Krok 1 2014 - anatomy
Krok 1   2014 - anatomyKrok 1   2014 - anatomy
Krok 1 2014 - anatomyEneutron
 
Krok 1 - 2013 Question Paper (General medicine)
Krok 1 - 2013 Question Paper (General medicine)Krok 1 - 2013 Question Paper (General medicine)
Krok 1 - 2013 Question Paper (General medicine)Eneutron
 
Krok 1 - 2014 (Path-Physiology)
Krok 1 - 2014 (Path-Physiology)Krok 1 - 2014 (Path-Physiology)
Krok 1 - 2014 (Path-Physiology)Eneutron
 
Krok 1 2014 - path physiology
Krok 1   2014 - path physiologyKrok 1   2014 - path physiology
Krok 1 2014 - path physiologyEneutron
 
Krok 1 - 2014 (Biology)
Krok 1 - 2014 (Biology)Krok 1 - 2014 (Biology)
Krok 1 - 2014 (Biology)Eneutron
 

What's hot (18)

Krok 1 - 2010 Question Paper (General medicine)
Krok 1 - 2010 Question Paper (General medicine)Krok 1 - 2010 Question Paper (General medicine)
Krok 1 - 2010 Question Paper (General medicine)
 
Krok 1 - 2008 Question Paper (General medicine)
Krok 1 - 2008 Question Paper (General medicine)Krok 1 - 2008 Question Paper (General medicine)
Krok 1 - 2008 Question Paper (General medicine)
 
Krok 1 - 2015 (Biochemistry)
Krok 1 - 2015 (Biochemistry)Krok 1 - 2015 (Biochemistry)
Krok 1 - 2015 (Biochemistry)
 
Krok 1 - 2014 (Biochemistry)
Krok 1 - 2014 (Biochemistry)Krok 1 - 2014 (Biochemistry)
Krok 1 - 2014 (Biochemistry)
 
Krok 1 - 2014 (Anatomy)
Krok 1 -  2014 (Anatomy)Krok 1 -  2014 (Anatomy)
Krok 1 - 2014 (Anatomy)
 
Krok 1 - 2015 (Physiology)
Krok 1 - 2015 (Physiology)Krok 1 - 2015 (Physiology)
Krok 1 - 2015 (Physiology)
 
Krok 1 - 2006 Question Paper (General medicine)
Krok 1 - 2006 Question Paper (General medicine)Krok 1 - 2006 Question Paper (General medicine)
Krok 1 - 2006 Question Paper (General medicine)
 
Krok 1 - 2012 Question Paper (General medicine)
Krok 1 - 2012 Question Paper (General medicine)Krok 1 - 2012 Question Paper (General medicine)
Krok 1 - 2012 Question Paper (General medicine)
 
Krok 1 - 2015 (Anatomy)
Krok 1 - 2015 (Anatomy)Krok 1 - 2015 (Anatomy)
Krok 1 - 2015 (Anatomy)
 
Krok 1 - 2011 Question Paper (General medicine)
Krok 1 - 2011 Question Paper (General medicine)Krok 1 - 2011 Question Paper (General medicine)
Krok 1 - 2011 Question Paper (General medicine)
 
Krok 1 - 2009 Question Paper (General medicine)
Krok 1 - 2009 Question Paper (General medicine)Krok 1 - 2009 Question Paper (General medicine)
Krok 1 - 2009 Question Paper (General medicine)
 
Krok 1 2014 - biochemistry
Krok 1   2014 - biochemistryKrok 1   2014 - biochemistry
Krok 1 2014 - biochemistry
 
Krok 1 - 2015 Base (General Medicine)
Krok 1 - 2015 Base (General Medicine)Krok 1 - 2015 Base (General Medicine)
Krok 1 - 2015 Base (General Medicine)
 
Krok 1 2014 - anatomy
Krok 1   2014 - anatomyKrok 1   2014 - anatomy
Krok 1 2014 - anatomy
 
Krok 1 - 2013 Question Paper (General medicine)
Krok 1 - 2013 Question Paper (General medicine)Krok 1 - 2013 Question Paper (General medicine)
Krok 1 - 2013 Question Paper (General medicine)
 
Krok 1 - 2014 (Path-Physiology)
Krok 1 - 2014 (Path-Physiology)Krok 1 - 2014 (Path-Physiology)
Krok 1 - 2014 (Path-Physiology)
 
Krok 1 2014 - path physiology
Krok 1   2014 - path physiologyKrok 1   2014 - path physiology
Krok 1 2014 - path physiology
 
Krok 1 - 2014 (Biology)
Krok 1 - 2014 (Biology)Krok 1 - 2014 (Biology)
Krok 1 - 2014 (Biology)
 

Viewers also liked

Krok 1 - 2014 Path-Anatomy Base (General Medicine)
Krok 1 - 2014 Path-Anatomy Base (General Medicine)Krok 1 - 2014 Path-Anatomy Base (General Medicine)
Krok 1 - 2014 Path-Anatomy Base (General Medicine)E_neutron
 
Krok 1 - 2014 Pharmacology Base (General Medicine)
Krok 1 - 2014 Pharmacology Base (General Medicine)Krok 1 - 2014 Pharmacology Base (General Medicine)
Krok 1 - 2014 Pharmacology Base (General Medicine)E_neutron
 
Krok 1 - 2014 Microbiology Base (General Medicine)
Krok 1 - 2014 Microbiology Base (General Medicine)Krok 1 - 2014 Microbiology Base (General Medicine)
Krok 1 - 2014 Microbiology Base (General Medicine)E_neutron
 
Krok 1 - 2014 Path-Physiology Base (General Medicine)
Krok 1 - 2014 Path-Physiology Base (General Medicine)Krok 1 - 2014 Path-Physiology Base (General Medicine)
Krok 1 - 2014 Path-Physiology Base (General Medicine)E_neutron
 
Bahan ajar Tekanan Osmotik
Bahan ajar Tekanan OsmotikBahan ajar Tekanan Osmotik
Bahan ajar Tekanan OsmotikShanty Stya
 

Viewers also liked (6)

Krok 1 - 2014 Path-Anatomy Base (General Medicine)
Krok 1 - 2014 Path-Anatomy Base (General Medicine)Krok 1 - 2014 Path-Anatomy Base (General Medicine)
Krok 1 - 2014 Path-Anatomy Base (General Medicine)
 
Krok 1 - 2014 Pharmacology Base (General Medicine)
Krok 1 - 2014 Pharmacology Base (General Medicine)Krok 1 - 2014 Pharmacology Base (General Medicine)
Krok 1 - 2014 Pharmacology Base (General Medicine)
 
Krok 1 - 2014 Microbiology Base (General Medicine)
Krok 1 - 2014 Microbiology Base (General Medicine)Krok 1 - 2014 Microbiology Base (General Medicine)
Krok 1 - 2014 Microbiology Base (General Medicine)
 
Krok 1 - 2014 Path-Physiology Base (General Medicine)
Krok 1 - 2014 Path-Physiology Base (General Medicine)Krok 1 - 2014 Path-Physiology Base (General Medicine)
Krok 1 - 2014 Path-Physiology Base (General Medicine)
 
Bahan ajar Tekanan Osmotik
Bahan ajar Tekanan OsmotikBahan ajar Tekanan Osmotik
Bahan ajar Tekanan Osmotik
 
About Osmosis
About OsmosisAbout Osmosis
About Osmosis
 

Similar to Krok 1 - 2014 Physiology Base (General Medicine)

Antiarrhytmics screening models.pptx
Antiarrhytmics screening models.pptxAntiarrhytmics screening models.pptx
Antiarrhytmics screening models.pptxsharanuGurikar
 
cholinergic drugs theory.pptx
cholinergic drugs theory.pptxcholinergic drugs theory.pptx
cholinergic drugs theory.pptxNagendraNayak12
 
CARDIOPULMONARY RESUSCITATION (CPR).pptx
CARDIOPULMONARY RESUSCITATION (CPR).pptxCARDIOPULMONARY RESUSCITATION (CPR).pptx
CARDIOPULMONARY RESUSCITATION (CPR).pptxssuser81b77c
 
Cholinomemtic drugs
Cholinomemtic drugsCholinomemtic drugs
Cholinomemtic drugshuma2012
 
Krok 1 - 2007 Question Paper (General medicine)
Krok 1 - 2007 Question Paper (General medicine)Krok 1 - 2007 Question Paper (General medicine)
Krok 1 - 2007 Question Paper (General medicine)Eneutron
 
Adrenalgland
AdrenalglandAdrenalgland
AdrenalglandMoH
 
Preclinical screening of anti anginals
Preclinical screening of anti anginalsPreclinical screening of anti anginals
Preclinical screening of anti anginalsDekollu Suku
 
شوک در جراحی.pptx
شوک در جراحی.pptxشوک در جراحی.pptx
شوک در جراحی.pptxbaharhoseini
 
Donation after brain death
Donation after brain deathDonation after brain death
Donation after brain deathVinodh Natarajan
 
Shock its pathopysiology and management
Shock its pathopysiology and managementShock its pathopysiology and management
Shock its pathopysiology and managementSHAKIL JAWED
 
Complications - spinal cord injury .pptx
Complications - spinal cord injury .pptxComplications - spinal cord injury .pptx
Complications - spinal cord injury .pptxDrJDP
 

Similar to Krok 1 - 2014 Physiology Base (General Medicine) (20)

Anticholinergics
Anticholinergics  Anticholinergics
Anticholinergics
 
2009
20092009
2009
 
2010
20102010
2010
 
ANS - Doc Rasha
ANS - Doc RashaANS - Doc Rasha
ANS - Doc Rasha
 
Antiarrhytmics screening models.pptx
Antiarrhytmics screening models.pptxAntiarrhytmics screening models.pptx
Antiarrhytmics screening models.pptx
 
cholinergic drugs theory.pptx
cholinergic drugs theory.pptxcholinergic drugs theory.pptx
cholinergic drugs theory.pptx
 
CARDIOPULMONARY RESUSCITATION (CPR).pptx
CARDIOPULMONARY RESUSCITATION (CPR).pptxCARDIOPULMONARY RESUSCITATION (CPR).pptx
CARDIOPULMONARY RESUSCITATION (CPR).pptx
 
Cholinomemtic drugs
Cholinomemtic drugsCholinomemtic drugs
Cholinomemtic drugs
 
Krok 1 - 2007 Question Paper (General medicine)
Krok 1 - 2007 Question Paper (General medicine)Krok 1 - 2007 Question Paper (General medicine)
Krok 1 - 2007 Question Paper (General medicine)
 
Adrenalgland
AdrenalglandAdrenalgland
Adrenalgland
 
Preclinical screening of anti anginals
Preclinical screening of anti anginalsPreclinical screening of anti anginals
Preclinical screening of anti anginals
 
شوک در جراحی.pptx
شوک در جراحی.pptxشوک در جراحی.pptx
شوک در جراحی.pptx
 
Brain death
Brain deathBrain death
Brain death
 
ECT .pptx
ECT .pptxECT .pptx
ECT .pptx
 
Donation after brain death
Donation after brain deathDonation after brain death
Donation after brain death
 
SHOCK
SHOCKSHOCK
SHOCK
 
Shock its pathopysiology and management
Shock its pathopysiology and managementShock its pathopysiology and management
Shock its pathopysiology and management
 
Electroconvulsive therapy
Electroconvulsive therapyElectroconvulsive therapy
Electroconvulsive therapy
 
Tests
TestsTests
Tests
 
Complications - spinal cord injury .pptx
Complications - spinal cord injury .pptxComplications - spinal cord injury .pptx
Complications - spinal cord injury .pptx
 

Recently uploaded

Call Girls Coimbatore Just Call 9907093804 Top Class Call Girl Service Available
Call Girls Coimbatore Just Call 9907093804 Top Class Call Girl Service AvailableCall Girls Coimbatore Just Call 9907093804 Top Class Call Girl Service Available
Call Girls Coimbatore Just Call 9907093804 Top Class Call Girl Service AvailableDipal Arora
 
Call Girls Bhubaneswar Just Call 9907093804 Top Class Call Girl Service Avail...
Call Girls Bhubaneswar Just Call 9907093804 Top Class Call Girl Service Avail...Call Girls Bhubaneswar Just Call 9907093804 Top Class Call Girl Service Avail...
Call Girls Bhubaneswar Just Call 9907093804 Top Class Call Girl Service Avail...Dipal Arora
 
Call Girls Faridabad Just Call 9907093804 Top Class Call Girl Service Available
Call Girls Faridabad Just Call 9907093804 Top Class Call Girl Service AvailableCall Girls Faridabad Just Call 9907093804 Top Class Call Girl Service Available
Call Girls Faridabad Just Call 9907093804 Top Class Call Girl Service AvailableDipal Arora
 
Call Girls Service Jaipur Grishma WhatsApp ❤8445551418 VIP Call Girls Jaipur
Call Girls Service Jaipur Grishma WhatsApp ❤8445551418 VIP Call Girls JaipurCall Girls Service Jaipur Grishma WhatsApp ❤8445551418 VIP Call Girls Jaipur
Call Girls Service Jaipur Grishma WhatsApp ❤8445551418 VIP Call Girls Jaipurparulsinha
 
Call Girls Service Surat Samaira ❤️🍑 8250192130 👄 Independent Escort Service ...
Call Girls Service Surat Samaira ❤️🍑 8250192130 👄 Independent Escort Service ...Call Girls Service Surat Samaira ❤️🍑 8250192130 👄 Independent Escort Service ...
Call Girls Service Surat Samaira ❤️🍑 8250192130 👄 Independent Escort Service ...CALL GIRLS
 
(Rocky) Jaipur Call Girl - 09521753030 Escorts Service 50% Off with Cash ON D...
(Rocky) Jaipur Call Girl - 09521753030 Escorts Service 50% Off with Cash ON D...(Rocky) Jaipur Call Girl - 09521753030 Escorts Service 50% Off with Cash ON D...
(Rocky) Jaipur Call Girl - 09521753030 Escorts Service 50% Off with Cash ON D...indiancallgirl4rent
 
Night 7k to 12k Navi Mumbai Call Girl Photo 👉 BOOK NOW 9833363713 👈 ♀️ night ...
Night 7k to 12k Navi Mumbai Call Girl Photo 👉 BOOK NOW 9833363713 👈 ♀️ night ...Night 7k to 12k Navi Mumbai Call Girl Photo 👉 BOOK NOW 9833363713 👈 ♀️ night ...
Night 7k to 12k Navi Mumbai Call Girl Photo 👉 BOOK NOW 9833363713 👈 ♀️ night ...aartirawatdelhi
 
Call Girls Jabalpur Just Call 9907093804 Top Class Call Girl Service Available
Call Girls Jabalpur Just Call 9907093804 Top Class Call Girl Service AvailableCall Girls Jabalpur Just Call 9907093804 Top Class Call Girl Service Available
Call Girls Jabalpur Just Call 9907093804 Top Class Call Girl Service AvailableDipal Arora
 
Call Girls Varanasi Just Call 9907093804 Top Class Call Girl Service Available
Call Girls Varanasi Just Call 9907093804 Top Class Call Girl Service AvailableCall Girls Varanasi Just Call 9907093804 Top Class Call Girl Service Available
Call Girls Varanasi Just Call 9907093804 Top Class Call Girl Service AvailableDipal Arora
 
Call Girls Kochi Just Call 9907093804 Top Class Call Girl Service Available
Call Girls Kochi Just Call 9907093804 Top Class Call Girl Service AvailableCall Girls Kochi Just Call 9907093804 Top Class Call Girl Service Available
Call Girls Kochi Just Call 9907093804 Top Class Call Girl Service AvailableDipal Arora
 
Call Girls Aurangabad Just Call 9907093804 Top Class Call Girl Service Available
Call Girls Aurangabad Just Call 9907093804 Top Class Call Girl Service AvailableCall Girls Aurangabad Just Call 9907093804 Top Class Call Girl Service Available
Call Girls Aurangabad Just Call 9907093804 Top Class Call Girl Service AvailableDipal Arora
 
Call Girls Siliguri Just Call 9907093804 Top Class Call Girl Service Available
Call Girls Siliguri Just Call 9907093804 Top Class Call Girl Service AvailableCall Girls Siliguri Just Call 9907093804 Top Class Call Girl Service Available
Call Girls Siliguri Just Call 9907093804 Top Class Call Girl Service AvailableDipal Arora
 
Call Girls Nagpur Just Call 9907093804 Top Class Call Girl Service Available
Call Girls Nagpur Just Call 9907093804 Top Class Call Girl Service AvailableCall Girls Nagpur Just Call 9907093804 Top Class Call Girl Service Available
Call Girls Nagpur Just Call 9907093804 Top Class Call Girl Service AvailableDipal Arora
 
Night 7k to 12k Chennai City Center Call Girls 👉👉 7427069034⭐⭐ 100% Genuine E...
Night 7k to 12k Chennai City Center Call Girls 👉👉 7427069034⭐⭐ 100% Genuine E...Night 7k to 12k Chennai City Center Call Girls 👉👉 7427069034⭐⭐ 100% Genuine E...
Night 7k to 12k Chennai City Center Call Girls 👉👉 7427069034⭐⭐ 100% Genuine E...hotbabesbook
 
Bangalore Call Girls Nelamangala Number 7001035870 Meetin With Bangalore Esc...
Bangalore Call Girls Nelamangala Number 7001035870  Meetin With Bangalore Esc...Bangalore Call Girls Nelamangala Number 7001035870  Meetin With Bangalore Esc...
Bangalore Call Girls Nelamangala Number 7001035870 Meetin With Bangalore Esc...narwatsonia7
 
♛VVIP Hyderabad Call Girls Chintalkunta🖕7001035870🖕Riya Kappor Top Call Girl ...
♛VVIP Hyderabad Call Girls Chintalkunta🖕7001035870🖕Riya Kappor Top Call Girl ...♛VVIP Hyderabad Call Girls Chintalkunta🖕7001035870🖕Riya Kappor Top Call Girl ...
♛VVIP Hyderabad Call Girls Chintalkunta🖕7001035870🖕Riya Kappor Top Call Girl ...astropune
 
The Most Attractive Hyderabad Call Girls Kothapet 𖠋 6297143586 𖠋 Will You Mis...
The Most Attractive Hyderabad Call Girls Kothapet 𖠋 6297143586 𖠋 Will You Mis...The Most Attractive Hyderabad Call Girls Kothapet 𖠋 6297143586 𖠋 Will You Mis...
The Most Attractive Hyderabad Call Girls Kothapet 𖠋 6297143586 𖠋 Will You Mis...chandars293
 
Top Rated Hyderabad Call Girls Erragadda ⟟ 6297143586 ⟟ Call Me For Genuine ...
Top Rated  Hyderabad Call Girls Erragadda ⟟ 6297143586 ⟟ Call Me For Genuine ...Top Rated  Hyderabad Call Girls Erragadda ⟟ 6297143586 ⟟ Call Me For Genuine ...
Top Rated Hyderabad Call Girls Erragadda ⟟ 6297143586 ⟟ Call Me For Genuine ...chandars293
 
Call Girls Cuttack Just Call 9907093804 Top Class Call Girl Service Available
Call Girls Cuttack Just Call 9907093804 Top Class Call Girl Service AvailableCall Girls Cuttack Just Call 9907093804 Top Class Call Girl Service Available
Call Girls Cuttack Just Call 9907093804 Top Class Call Girl Service AvailableDipal Arora
 
Call Girls Dehradun Just Call 9907093804 Top Class Call Girl Service Available
Call Girls Dehradun Just Call 9907093804 Top Class Call Girl Service AvailableCall Girls Dehradun Just Call 9907093804 Top Class Call Girl Service Available
Call Girls Dehradun Just Call 9907093804 Top Class Call Girl Service AvailableDipal Arora
 

Recently uploaded (20)

Call Girls Coimbatore Just Call 9907093804 Top Class Call Girl Service Available
Call Girls Coimbatore Just Call 9907093804 Top Class Call Girl Service AvailableCall Girls Coimbatore Just Call 9907093804 Top Class Call Girl Service Available
Call Girls Coimbatore Just Call 9907093804 Top Class Call Girl Service Available
 
Call Girls Bhubaneswar Just Call 9907093804 Top Class Call Girl Service Avail...
Call Girls Bhubaneswar Just Call 9907093804 Top Class Call Girl Service Avail...Call Girls Bhubaneswar Just Call 9907093804 Top Class Call Girl Service Avail...
Call Girls Bhubaneswar Just Call 9907093804 Top Class Call Girl Service Avail...
 
Call Girls Faridabad Just Call 9907093804 Top Class Call Girl Service Available
Call Girls Faridabad Just Call 9907093804 Top Class Call Girl Service AvailableCall Girls Faridabad Just Call 9907093804 Top Class Call Girl Service Available
Call Girls Faridabad Just Call 9907093804 Top Class Call Girl Service Available
 
Call Girls Service Jaipur Grishma WhatsApp ❤8445551418 VIP Call Girls Jaipur
Call Girls Service Jaipur Grishma WhatsApp ❤8445551418 VIP Call Girls JaipurCall Girls Service Jaipur Grishma WhatsApp ❤8445551418 VIP Call Girls Jaipur
Call Girls Service Jaipur Grishma WhatsApp ❤8445551418 VIP Call Girls Jaipur
 
Call Girls Service Surat Samaira ❤️🍑 8250192130 👄 Independent Escort Service ...
Call Girls Service Surat Samaira ❤️🍑 8250192130 👄 Independent Escort Service ...Call Girls Service Surat Samaira ❤️🍑 8250192130 👄 Independent Escort Service ...
Call Girls Service Surat Samaira ❤️🍑 8250192130 👄 Independent Escort Service ...
 
(Rocky) Jaipur Call Girl - 09521753030 Escorts Service 50% Off with Cash ON D...
(Rocky) Jaipur Call Girl - 09521753030 Escorts Service 50% Off with Cash ON D...(Rocky) Jaipur Call Girl - 09521753030 Escorts Service 50% Off with Cash ON D...
(Rocky) Jaipur Call Girl - 09521753030 Escorts Service 50% Off with Cash ON D...
 
Night 7k to 12k Navi Mumbai Call Girl Photo 👉 BOOK NOW 9833363713 👈 ♀️ night ...
Night 7k to 12k Navi Mumbai Call Girl Photo 👉 BOOK NOW 9833363713 👈 ♀️ night ...Night 7k to 12k Navi Mumbai Call Girl Photo 👉 BOOK NOW 9833363713 👈 ♀️ night ...
Night 7k to 12k Navi Mumbai Call Girl Photo 👉 BOOK NOW 9833363713 👈 ♀️ night ...
 
Call Girls Jabalpur Just Call 9907093804 Top Class Call Girl Service Available
Call Girls Jabalpur Just Call 9907093804 Top Class Call Girl Service AvailableCall Girls Jabalpur Just Call 9907093804 Top Class Call Girl Service Available
Call Girls Jabalpur Just Call 9907093804 Top Class Call Girl Service Available
 
Call Girls Varanasi Just Call 9907093804 Top Class Call Girl Service Available
Call Girls Varanasi Just Call 9907093804 Top Class Call Girl Service AvailableCall Girls Varanasi Just Call 9907093804 Top Class Call Girl Service Available
Call Girls Varanasi Just Call 9907093804 Top Class Call Girl Service Available
 
Call Girls Kochi Just Call 9907093804 Top Class Call Girl Service Available
Call Girls Kochi Just Call 9907093804 Top Class Call Girl Service AvailableCall Girls Kochi Just Call 9907093804 Top Class Call Girl Service Available
Call Girls Kochi Just Call 9907093804 Top Class Call Girl Service Available
 
Call Girls Aurangabad Just Call 9907093804 Top Class Call Girl Service Available
Call Girls Aurangabad Just Call 9907093804 Top Class Call Girl Service AvailableCall Girls Aurangabad Just Call 9907093804 Top Class Call Girl Service Available
Call Girls Aurangabad Just Call 9907093804 Top Class Call Girl Service Available
 
Call Girls Siliguri Just Call 9907093804 Top Class Call Girl Service Available
Call Girls Siliguri Just Call 9907093804 Top Class Call Girl Service AvailableCall Girls Siliguri Just Call 9907093804 Top Class Call Girl Service Available
Call Girls Siliguri Just Call 9907093804 Top Class Call Girl Service Available
 
Call Girls Nagpur Just Call 9907093804 Top Class Call Girl Service Available
Call Girls Nagpur Just Call 9907093804 Top Class Call Girl Service AvailableCall Girls Nagpur Just Call 9907093804 Top Class Call Girl Service Available
Call Girls Nagpur Just Call 9907093804 Top Class Call Girl Service Available
 
Night 7k to 12k Chennai City Center Call Girls 👉👉 7427069034⭐⭐ 100% Genuine E...
Night 7k to 12k Chennai City Center Call Girls 👉👉 7427069034⭐⭐ 100% Genuine E...Night 7k to 12k Chennai City Center Call Girls 👉👉 7427069034⭐⭐ 100% Genuine E...
Night 7k to 12k Chennai City Center Call Girls 👉👉 7427069034⭐⭐ 100% Genuine E...
 
Bangalore Call Girls Nelamangala Number 7001035870 Meetin With Bangalore Esc...
Bangalore Call Girls Nelamangala Number 7001035870  Meetin With Bangalore Esc...Bangalore Call Girls Nelamangala Number 7001035870  Meetin With Bangalore Esc...
Bangalore Call Girls Nelamangala Number 7001035870 Meetin With Bangalore Esc...
 
♛VVIP Hyderabad Call Girls Chintalkunta🖕7001035870🖕Riya Kappor Top Call Girl ...
♛VVIP Hyderabad Call Girls Chintalkunta🖕7001035870🖕Riya Kappor Top Call Girl ...♛VVIP Hyderabad Call Girls Chintalkunta🖕7001035870🖕Riya Kappor Top Call Girl ...
♛VVIP Hyderabad Call Girls Chintalkunta🖕7001035870🖕Riya Kappor Top Call Girl ...
 
The Most Attractive Hyderabad Call Girls Kothapet 𖠋 6297143586 𖠋 Will You Mis...
The Most Attractive Hyderabad Call Girls Kothapet 𖠋 6297143586 𖠋 Will You Mis...The Most Attractive Hyderabad Call Girls Kothapet 𖠋 6297143586 𖠋 Will You Mis...
The Most Attractive Hyderabad Call Girls Kothapet 𖠋 6297143586 𖠋 Will You Mis...
 
Top Rated Hyderabad Call Girls Erragadda ⟟ 6297143586 ⟟ Call Me For Genuine ...
Top Rated  Hyderabad Call Girls Erragadda ⟟ 6297143586 ⟟ Call Me For Genuine ...Top Rated  Hyderabad Call Girls Erragadda ⟟ 6297143586 ⟟ Call Me For Genuine ...
Top Rated Hyderabad Call Girls Erragadda ⟟ 6297143586 ⟟ Call Me For Genuine ...
 
Call Girls Cuttack Just Call 9907093804 Top Class Call Girl Service Available
Call Girls Cuttack Just Call 9907093804 Top Class Call Girl Service AvailableCall Girls Cuttack Just Call 9907093804 Top Class Call Girl Service Available
Call Girls Cuttack Just Call 9907093804 Top Class Call Girl Service Available
 
Call Girls Dehradun Just Call 9907093804 Top Class Call Girl Service Available
Call Girls Dehradun Just Call 9907093804 Top Class Call Girl Service AvailableCall Girls Dehradun Just Call 9907093804 Top Class Call Girl Service Available
Call Girls Dehradun Just Call 9907093804 Top Class Call Girl Service Available
 

Krok 1 - 2014 Physiology Base (General Medicine)

  • 1. Krok 1 Medicine 4.0 Нормальна фізіологія 1 Patient with hypersecretion of the gastric juices was recomended to exclude from the diet concentrated bouillons and vegetable decoctions because of their stimulation of gastric secretion. What is dominating mechanism of stimulation of secretion in this case? A Stimulation of gastrin production by G-cells * B Irritation of taste receptors C Irritation of mechanoreceptors of the oral cavity D Irritation of mechanoreceptors of the stomach E Stimulation of excretion of secretin in the duodenum 2 Person felt thirsty after staying in heat for a long time. Signals of what receptors caused it first of all? A Osmoreceptors of hypothalamus * B Sodium receptors of hypothalamus C Osmoreceptorsof the liver D Glucoreceptors of hypothalamus E Baroreceptors of aortic arch 3 While emotional excitement the heart rate in a 30-year-old person run up to 112 Bpm. What part of the conducting system of the heart caused it? A Synoatrial node * B Purkinje's fibers C His bundle branches D Intraventricular node E His bundle 4 A 38-year-old woman was admitted to the admission-diagnostic department with uterine bleeding. What are the most likely changes of blood? A Reduction of haematocrite rate * B Increase of haematocrite rate C Leukopenia D Leucocytosis E Polycythemia
  • 2. 5 Due to action of electric current on the exitable cell there appeared depolarization of it's membrane. Movement of what ions through the membrane caused depolarisation? A Na+ B НСО3 - CСа2+ D Сl- EК+ 6 On experiment on the dog the peripheral part of nervus vagus of the neck was irritated. What changes of the heart function would be observed? ADecreased contraction rate * BIncreased contraction force CIncreased atrioventricular conduction D Increased contraction force and rate EIncreased myocardial excitability 7 Power inputs of a boy increased from 500 to 2000 kJ pro hour. What can be the cause of it? APhysical exercise * BRaise of outer temperatute CMental activity D Food intake ETransition from sleep to wakefulness 8 In a young man during exercise, the minute oxygen uptake and carbon dioxide emission equalled to 1000 ml. What substrates are oxidized in the cells of his body? ACarbohydrates * BProteins CFats D Carbohydrates and fats ECarbohydrates and proteins 9 In the experiment on the animal the part of the cerebral cortex hemispheres was removed. It caused elimination of previously formed conditioned reflex to the light irritation. What part ot the cortex was removed? AOccipital cortex * BPrecentral convolution CPostcentral convolution
  • 3. D Limbic cortex ETemporal lobe 10 Inhibition of alpha-motoneuron of the extensor muscles was noticed after stimulation of alpha-motoneuron of the flexor muscles during the experiment on the spinal column. What type of inhibition can this process cause? AReciprocal * BPresynaptic CDepolarizational D Recurrent ELateral 11 Respiratory coefficient was studied in the patient who strictly kept certain diet for 10 days. It was determined that it is 1. What diet does the patient follow? AWith domination of carbohydrates * BWith domination of proteins and fat CWith domination of fat and carbohydrates D Mixed EWith domination of proteins and carbohydrates 12 A sportsman spontaneously held breath for 40 seconds, which resulted in an increase in heart rate and systemic arterial pressure. Changes of these indicators are due to activation of the following regulatory mechanisms: AUnconditioned sympathetic reflexes * BUnconditioned parasympathetic reflexes CConditioned sympathetic reflexes D Conditioned parasympathetic reflexes E- 13 Punctata hemorrhage was found out in the patient after application of a tourniquet. With disfunction of what blood cells is it connected? APlatelets * BEosinophiles CMonocytes D Lymphocytes ENeutrophiles
  • 4. 14 Students who are taking examinations often have dry mouth. The mechanism that causes this state is the realization of the following reflexes: AConditioned sympathetic * BUnconditioned parasympathetic CConditioned parasympathetic D Unconditioned sympathetic EUnconditioned peripheral 15 Middle part of cochlear of internal ear was destroyed in animal while experiment. It will cause abnormalities of the sound perception of the following frequencies: AMiddle * BLow CHigh D High and low ENo abnormalities 16 The temperature of the ambient environment is 38oC and relative air humidity is 50%. What ways of heat emission provide maintaining a constant temperature of the human body? AEvaporation * BRadiation CHeat conduction D Convection EConvection and conduction 17 The minute blood volume in a patient with transplanted heart has increased as a result of physical activity. What regulative mechanism is responsible for these changes? ACatecholamines * BSympathetic unconditioned reflexes CParasympathetic unconditioned reflexes D Sympathetic conditioned reflexes EParasympathetic conditioned reflexes 18 Isolated muscle of a frog is rhythmically irritated with electric impulses. Every next impulse is in a period of relaxation from the previus contraction. What contraction of the muscle appears? AWaved tetanus * BSingle CAsynchronous
  • 5. D Continuous(smooth) tetanus ETonic 19 A 30 year old woman has subnormal concentration of enzymes in the pancreatic juice. This might be caused by the hyposecretion of the following gastrointestinal hormone: ACholecystokinin-pancreozymin * BSomatostatin CSecretin D Gastro-inhibiting peptide EVaso-intestinal peptide 20 A patient has a trauma of sternocleidomastoid muscle. This caused a decrease in value of the following indicator of external respiration: AInspiratory reserve volume * BExpiratory reserve volume CRespiratory capacity D Residual volume EFunctional residual lung capacity 21 A man has normal sensitivity of his finger skin, however he doesn’t sense his wedding ring around the finger. What process induced by wearing of the ring has caused this phenomenon? AReceptor adaptation * BDevelopment of the fibrous tissue CAbnormality of the epidermis structure D Impaired circulation EAbnormality of the receptor structure 22 An aged man had raise of arterial pressure under a stress. It was caused by activation of: ASympathoadrenal system * BParasympathetic nucleus of vagus CFunctions of thyroid gland D Functions of adrenal cortex EHypophysis function 23 A month after surgical constriction of rabbit's renal artery the considerable increase of systematic arterial pressure was observed. What of the following regulation mechanisms
  • 6. caused the animal's pressure change? AAngiotensin-II * BVasopressin CAdrenaline D Noradrenaline ESerotonin 24 A child has abnormal formation of tooth enamel and dentin as a result of low concentration of calcium ions in blood. Such abnormalities might be caused by deficiency of the following hormone: AParathormone * BThyrocalcitonin CThyroxin D Somatotropic hormone ETriiodothyronine 25 A sportsman was examined after an intensive physical activity. The examination revealed disorder of movement coordination but the force of muscle contractions remained the same. It can be explained by retarded speed of excitement conduction through: ACentral synapses * BNeuromuscular synapses CEfferent nerves D Afferent nerves EConduction tracts 26 After a long training session a sportsman has developed fatigue accompanied by abrupt performance decrement. What link of the reflex arch was the fatigue initiated in? ANerve centres * BAfferent conductor CReceptors D Efferent conductor EMuscles 27 ECG study showed that the $T$-waves were positive in the standard extremity leads, their amplitude and duration were normal. The right conclusion would be that the following process runs normally in the heart ventricles: ARepolarization * BDepolarization
  • 7. CExcitement D Contraction ERelaxation 28 Blood minute volume of a 30 year old woman at rest is 5 l/m. What blood volume is pumped through the pulmonary vessels per minute? A5 l * B3,75 l C2,5 l D 2,0 l E1,5 l 29 As a result of long-term starvation the glomerular filtration of a man was accelerated by 20%. The most probable cause of filtration changes under such conditions is: AFall of oncotic pressure of blood plasma * BRise of systemic arterial pressure CIncreased permeability of renal filter D Growth of filtration coefficient EIncrease of renal plasma flow 30 In course of an experiment a skeletal muscle is being stimulated by a series of electric impulses. What type of muscle contraction will arise, if every subsequent impulse comes in the period of shortening of the previous single muscle contraction? AHolotetanus * BPartial tetanus CAsynchronous tetanus D A series of single contractions EMuscle contracture 31 A patient under test was subjected to a moderate physical stress. His minute blood volume amounted 10 l/min. What blood volume was pumped through his lung vessels every minute? A10 l/min * B5 l/min C4 l/min D 6 l/min E7 l/min
  • 8. 32 A patient presents with the following motor activity disturbances: tremor, ataxia and asynergia movements, dysarthria. The disturbances are most likely to be localized in: ACerebellum * BBasal ganglions CLimbic system D Brainstem EMedulla oblongata 33 A man has a considerable decrease in diuresis as a result of 1,5 l blood loss. The primary cause of such diuresis disorder is the hypersecretion of the following hormone: AVasopressin * BCorticotropin CNatriuretic D Cortisol EParathormone 34 An animal experiment is aimed at studying the cardiac cycle. All the heart valves are closed. What phase of the cycle is characterized by this status? AIsometric contraction * BAsynchronous contraction CProtodiastolic period D Rapid filling EReduced filling 35 While shifting the gaze to the closely situated object the refracting power of eye's optical mediums will increase by 10 diopters. It results from changing of such eye structure: ALens * BCornea CVitreous body D Liquid of the anterior chamber of eye EMuscle that dilatates pupil 36 Spasm of smooth muscle of bronchi developed in the patient. Usage of activators of what membrane cytoreceptors is fisiologically valid to decrease attack? Aβ-adrenoreceptors * Bα-аdrenoreceptors Cα- та β-аdrenoreceptors
  • 9. D Н-cholinoreceptors EМ-cholinoreceptors 37 On examination of the person it was revealed that minute volume of heart is 3500mL, systolic volume is 50 mL. What is the frequency of cardiac contraction? A70 bpm * B60 bpm C50 bpm D 80 bpm E90 bpm 38 Due to activation of ion channels of external membrane of excitable cell it's rest potential has significantly increased. What channels were activated? APotassium channels * BNatrium channels CFast calcium channels D Slow calcium channels ENatrium and calcium channels 39 The ventral roots of 5 frontal segment of spinal cord were cut during experiment in the animal. What changes will take place in the innervation region? ALoss of movements * BLoss of touch sensitivity CLoss of temperature sensitivity D Loss of proprioceptive sensitivity EHypersensitivity 40 Glomerular filtration rate (GFR) increased for 20% due to prolonged starvation of the person. The most likely cause of filtration changes under this conditions is: ADecrease of oncotic pressure of blood plasma * BIncrease of systemic blood pressure CIncrease of penetration of the renal filter D Increase of filtration coefficient EIncrease of renal plasma stream 41 A patient has a transverse disruption of spinal cord below the IV thoracic segment. What
  • 10. changes of respiration will it cause? ARespiration will stay unchanged * BRespiration will stop CRespiration will become less frequent D Respiration will become deeper ERespiration will become more frequent 42 A lightly dressed man is standing in a room, air temperature is +$14^0C$, windows and doors are closed. In what way does he emit heat the most actively? AHeat radiation * BHeat conduction CConvection D Evaporation EPerspiration 43 ECG of a patient with hyperfunction of thyroid gland showed heart hurry. It is indicated by depression of the following ECG element: AR-R interval * BP-Q segment CP-Q interval D P-T interval EQRS complex 44 A peripheral segment of vagus nerve on a dog's neck was being stimulated in course of an experiment. The following changes of cardiac activity could be meanwhile observed: AHeart rate fall * BHeart hurry CEnhancement of atrioventricular conduction D Heart rate and heart force amplification EIncreased excitability of myocardium 45 ECG of a patient shows prolongation of T-wave. This is caused by deceleration in ventricles of: ARepolarization * BDepolarization and repolarization CDepolarization D Contraction ERelaxation
  • 11. 46 In a healthy adult speed of the excitement conduction through the atrioventricular node is 0,02-0,05 m/sec. Atrioventricular delay enables: ASequence of atrial and ventricular contractions * BSimultaneity of both atria contractions CSimultaneity of both ventricles contractions D Sufficient force of atrial contractions ESufficient force of ventricular contractions 47 A 2 y.o. child has convulsions as a result of lowered concentration of calcium ions in blood plasma. It is caused by reduced function of: AParathyroid glands * BHypophysis CAdrenal cortex D Pineal gland EThymus 48 During preparation of a patient to a heart surgery it was necessary to measure pressure in heart chambers. In one of them pressure varied from 0 mm Hg up to 120 mm Hg within one cardiac cycle. What heart chamber is it? ALeft ventricle * BRight ventricle CRight atrium D Left atrium E- 49 Heart rate of a man permanently equals 40 beats pro minute. What is the pacemaker? AAtriventricular node * BSinoatrial node CHis' bundle D His' bundle branches EPurkinje's fibers 50 Stimulation of an excitable cell by the electric current has led to the depolarization of its membrane. The depolarization has been caused mainly by the following ions penetrating into the cell through its membrane:
  • 12. ANa+ BHCO3- CCa2+ D Cl- EK+ 51 Parents of a 10 y.o. boy consulted a doctor about extension of hair-covering, growth of beard and moustache, low voice. Intensified secretion of which hormone must be assumed? AOf testosterone * BOf somatotropin COf oestrogen D Of progesterone EOf cortisol 52 Lung ventilation in a person is increased as a result of physical activity. Which of the following indices of the external respiration is much higher than in a state of rest? ARespiratory volume * BVital capacity of lungs CInspiratory reserve volume D Expiratory reserve volume ETotal lung capacity 53 A man took a quiet expiration. Name an air volume that is meanwhile contained in his lungs: AFunctional residual capacity * BResidual volume CExpiratory reserve volume D Respiratory volume EVital lung capacity 54 Examination of an isolated cardiomyocyte revealed that it didn't generate excitation impulses automatically. This cardiomyocyte was obtained from: AVentricles * BSinoatrial node CAtrioventricular node D His' bundle EPurkinje's fibers
  • 13. 55 Examination of a man established that cardiac output equaled 3500 ml, systolic output - 50 ml. What is the man's heart rate pro minute? A70 * B60 C50 D 80 E90 56 As a result of continuous starvation the glomerular filtration rate has increased by 20%. The most probable cause of the glomerular filtration alteration under the mentioned conditions is: ADecrease in the oncotic pressure of blood plasma * BIncrease in the systemic arterial pressure CIncrease in the permeability of the renal filter D Increase of the filtartion quotient EIncrease of the renal blood flow 57 A man who went for a ride on a roundabout had amplification of heart rate, sweating and nausea. What receptors stimulation is it primarily connected with? AVestibular * BProprioceptors CTactors D Auditory EVisual 58 A man's intrapleural pressure is being measured. In what phase did the man hold his breath, if his pressure is 7,5 cm Hg? AQuiet inspiration * BQuiet expiration CForced inspiration D Forced expiration E- 59 Atria of an experimental animal were superdistended by blood that resulted in decreased reabsorption of Na+ and water in renal tubules. This can be explained by the influence of the following factor upon kidneys: ANatriuretic hormone * BAldosterone
  • 14. CRenin D Angiotensin EVasopressin 60 A middle-aged man went to a foreign country because he had been offered a job there. However he had been unemployed for quite a long time. What endocrine glands were exhausted most of all in this man? AAdrenal glands * BParathyroid glands CSeminal glands D Substernal gland EThyroid gland 61 A 42 year old patient complains of pain in the epigastral area, vomiting; vomit masses have the colour of "coffee-grounds", the patient has also melena. Anamnesis records gastric ulcer. Blood formula: erythrocytes - 2,8 *10 12/l, leukocytes – 8 *10 9/l, Hb- 90 g/l. What complication is it? AHaemorrhage * BPenetration CPerforation D Canceration EPyloric stenosis 62 A human body cools in water much faster that in the air. What way of heat emission in water is much more efficient? AHeat conduction * BConvection CHeat radiation D Sweat evaporation E- 63 As a result of spinal-cord trauma a 33 y.o. man has a disturbed pain and temperature sensitivity that is caused by damage of the following tract: ASpinothalamic * BMedial spinocortical CPosterior spinocerebellar D Lateral spinocortical EAnterior spinocerebellar
  • 15. 64 After a surgery a 36-year-old woman was given an intravenous injection of concentrated albumin solution. This has induced intensified water movement in the following direction: AFrom the intercellular fluid to the capillaries * BFrom the intercellular fluid to the cells CFrom the cells to the intercellular fluid D From the capillaries to the intercellular fluid ENo changes of water movement will be observed 65 A clinic observes a 49 year old patient with significant prolongation of coagulation time, gastrointestinal haemorrhages, subcutaneous hematomas. These symptoms might be explained by the deficiency of the following vitamin: AK * BB 1 CB6 D H EE 66 Examination of a patient revealed hyperkaliemia and hyponatremia. Low secretion of which hormone may cause such changes? AAldosteron * BVasopressin CCortisol D Parathormone ENatriuretic 67 Examination of a 43 y.o. anephric patient revealed anemia symptoms. What is the cause of these symptoms? AReduced synthesis of erythropoietins * BEnhanced destruction of erythrocytes CIron deficit D Vitamin B12 deficit EFolic acid deficit 68 While determining power inputs of a patient’s organism it was established that the respiratory coefficient equaled 1,0. This means that in the cells of the patient the following substances are mainly oxidized:
  • 16. ACarbohydrates * BProteins CFats D Proteins and carbohydrates ECarbohydrates and fats 69 A patient has a disturbed absorbtion of fat hydrolysates. It might have been caused by a deficit in the small intestine cavity: AOf bile acids * BOf bile pigments COf lipolytic enzymes D Of sodium ions EOf liposoluble vitamins 70 Inhabitants of territories with cold climate have high content of an adaptive thermoregulatory hormone. What hormone is meant? AThyroxin * BInsulin CGlucagon D Somatotropin ECortisol 71 A concentrated solution of sodium chloride was intravenously injected to an animal. This caused decreased reabsorption of sodium ions in the renal tubules. It is the result of the following changes of hormonal secretion: AAldosterone reduction * BAldosterone increase CVasopressin reduction D Vasopressin increase EReduction of atrial natriuretic factor 72 People adapted to high external temperatures have such pecularity: profuse sweating isn't accompanied by loss of large volumes of sodium chloride. This is caused by the effect of the following hormone upon the perspiratory glands: AAldosterone * BVasopressin CCortisol D Tgyroxin
  • 17. ENatriuretic 73 The processes of heat transfer in a naked person at room temperature have been studied. It was revealed that under these conditions the greatest amount of heat is transferred by: AHeat radiation * BHeat conduction CConvection D Evaporation E- 74 During an experiment the dorsal roots of the spinal cord of an animal have been cut. What changes will be observed in the innervation zone? ASensitivity loss * BLoss of motor functions CDecrease in muscle tone D Increase in muscle tone ESensitivity loss and loss of motor functions 75 As a result of destruction of certain brainstem structures an animal has lost its orientative reflexes in response to strong photic stimuli. What structures were destroyed? AAnterior tubercles of quadrigeminal plate * BPosterior tubercles of quadrigeminal plate CRed nuclei D Vestibular nuclei EBlack substance 76 As a result of damage to certain structures of brainstem an animal lost orientation reflexes. What structures were damaged? AQuadritubercular bodies * BMedial nuclei of reticular formation CRed nuclei D Vestibular nuclei EBlack substance 77 Urine analysis has shown high levels of protein and erythrocytes in urine. This can be caused by the following:
  • 18. ARenal filter permeability * BEffective filter pressure CHydrostatic blood pressure in glomerular capillaries D Hydrostatic primary urine pressure in capsule EOncotic pressure of blood plasma 78 Osmotic pressure of a man's blood plasma is 350 mosmole/l (standard pressure is 300 mosmole/l). First of all it will result in high secretion of the following hormone: AVasopressin * BAldosteron CCortisol D Adrenocorticotropin ENatriuretic 79 After a craniocerebral injury a patient is unable to recognize objects by touch. What part of brain has been damaged? APostcentral gyrus * BOccipital lobe CTemporal lobe D Precentral gyrus ECerebellum 80 A hypertensive glucose solution was introduced to a patient. It will intensify water movement: AFrom the cells to the intercellular liquid * BFrom the intercellular liquid to the capillaries CFrom the intercellular liquid to the cells D From the capillaries to the intercellular liquid EThere will be no changes of water movement 81 To prevent long-term effects of 4-day malaria a 42-year-old patient was prescribed primaquine. On the 3-rd day from the begin of treatment there appeared stomach and heart pains, dyspepsia, general cyanosis, hemoglobinuria. What caused side effects of the preparation? AGenetic insufficiency of glucose 6-phosphate dehydrogenase * BCumulation of the preparation CDecreased activity of microsomal liver enzymes D Delayed urinary excretion of the preparation EDrug potentiation by other preparations
  • 19. 82 According to audiometry data a patient has a disturbed perception of medium-frequency sounds. It might have been caused by a damage of: AMiddle part of helix * BCochlear nuclei CSpiral ganglion D Quadritubercular structure ELateral geniculate bodies 83 A 17-year-old boy fell seriously ill, body temperature rose up to 38,5 oС, there is cough, rhinitis, lacrimation, nasal discharges. What kind of inflammation is it? ACatarrhal inflammation * BSerous inflammation CFibrinous inflammation D Suppurative inflammation EHemorrhagic inflammation 84 A patient with disturbed cerebral circulation has problems with deglutition. What part of brain was damaged? ABrainstem * BCervical part of spinal cord CForebrain D Interbrain EMidbrain 85 A patient who has been treated with diazepam on account of neurosis complains of toothache. Doctor administered him an analgetic, but its dose was lower than average therapeutic dose. What phenomenon did the doctor take into account while prescribing the patient an underdose? APotentiation * BSummation CCumulation D Drug dependence ETolerance 86 Long-term starvation cure of a patient resulted in diminished ratio of albumines and globulines in plasma. What of the following will be result of these changes? AIncrease of ESR *
  • 20. BDecrease of ESR CIncrease of hematocrit D Decrease of hematocrit EHypercoagulation 87 A patient has a decreased vasopressin synthesis that causes polyuria and as a result of it evident organism dehydratation. What is the mechanism of polyuria development? AReduced tubular reabsorption of water * BReduced tubular reabsorption of Na ions CReduced tubular reabsorption of protein D Reduced glucose reabsorption EAcceleration of glomerular filtration 88 A 35 year old man consulted a dentist about reduced density of dental tissue, high fragility of teeth during eating solid food. This patient suffers the most probably from the deficiency of the following mineral element: ACalcium * BPotassium CSodium D Magnesium EIron 89 A patient is 44 years old. Laboratory examination of his blood revealed that content of proteins in plasma was 40 g/l. What influence will be exerted on the transcapillary water exchange? AFiltration will be increased, reabsorption - decreased * BBoth filtration and reabsorption will be increased CBoth filtration and reabsorption will be decreased D Filtration will be decreased, reabsorption - increased EExchange will stay unchanged 90 A patient has a critical impairment of protein, fat and hydrocarbon digestion. Most likely it has been caused by low secretion of the following digestive juice: APancreatic juice * BSaliva CGastric juice D Bile EIntestinal juice
  • 21. 91 After destruction of CNS structures an animal lost orientative reflexes. What structure was destroyed? AQuadrigeminal plate * BRed nucleus CLateral vestibular nuclei D Black substance EMedial reticular nuclei 92 An isolated cell of human heart automatically generates excitation impulses with frequency 60 times pro minute. What structure does this cell belong to? ASinoatrial node * BAtrium CVentricle D Atrioventricular node EHis' bundle 93 A patient has a traumatic injury of sternocleidomastoid muscle. This has resulted in a decrease in the following value: AInspiratory reserve volume * BExpiratory reserve volume CRespiratory volume D Residual volume EFunctional residual lung capacity 94 Examination of a patient revealed a strong, balanced, inert type of higher nervous activity according to Pavlov. What temperament type does the patient have (according to Hippocrates classification)? APhlegmatic * BSanguine CCholeric D Melancholic E- 95 The receptors under study provide transfer of information to the cortex without thalamic involvement. Specify these receptors: AOlfactory *
  • 22. BTactile CGustatory D Visual EAuditory 96 Examination of a patient revealed overgrowth of facial bones and soft tissues, tongue enlargement, wide interdental spaces in the enlarged dental arch. What changes of the hormonal secretion are the most likely? AHypersecretion of the somatotropic hormone * BHyposecretion of the somatotropic hormone CHypersecretion of insulin D Hyposecretion of thyroxin EHyposecretion of insulin 97 A patient has a haemorrhage into the posterior central gyrus. What type of sensitivity on the opposite side will be disturbed? ASkin and proprioceptive * BVisual CAuditory D Olfactory EAuditory and visual 98 A 32-year-old patient consulted a doctor about the absence of lactation after parturition. Such disorder might be explained by the deficit of the following hormone: AProlactin * BSomatotropin CVasopressin D Thyrocalcitonin EGlucagon 99 During influenza epidemic 40% of pupils who didn't go in for sports were affected by the disease, and among the pupils who regularly did physical exercises this index was only 20%. What adaptative mechanisms determined such a low sickness rate of pupils participating in the sports? ACross adaptation * BSpecific adaptation CPhysiological adaptation D Biochemical adaptation
  • 23. EGenetic adaptation 100 A 60 year old patient was found to have a dysfunction of main digestive enzyme of saliva. This causes the disturbance of primary hydrolysis of: ACarbohydrates * BFats CProteins D Cellulose ELactose 101 A 49 year old woman spent a lot of time standing. As a result of it she got leg edema. What is the most likely cause of the edema? AIncrease in hydrostatic pressure of blood in veins * BDecrease in hydrostatic pressure of blood in veins CDecrease in hydrostatic pressure of blood in arteries D Increase in oncotic pressure of blood plasma EIncrease in systemic arterial pressure 102 A patient presented to a hospital with complaints about quick fatigability and significant muscle weakness. Examination revealed an autoimmune disease that causes functional disorder of receptors in the neuromuscular synapses. This will result in the disturbed activity of the following mediator: AAcetylcholine * BNoradrenaline CDopamine D Serotonin EGlycine 103 A 30-year-old woman was diagnosed with insufficiency of exocrinous function of pancreas. Hydrolisis of what nutrients will be disturbed? AProteins, fats, carbohydrates * BProteins, fats CProteins, carbohydrates D Fats, carbohydrates EProteins 104
  • 24. During an animal experiment, surgical damage of certain brain structures has caused deep prolonged sleep. What structure is most likely to cause such condition, if damaged? AReticular formation * BBasal ganglion CRed nuclei D Hippocampus ECerebral cortex 105 Short-term physical activity resulted in reflex amplification of heart rate and raise of systemic arterial pressure. What receptors activation was the main cause of pressor reflex realization? AProprioreceptors of active muscles * BVascular chemoreceptors CVascular volume receptors D Vascular baroceptors EHypothalamus thermoreceptors 106 In course of an experiment a skeletal muscle is being stimulated by a series of electric impulses. What type of muscle contraction will arise, if every subsequent impulse comes in the period of relaxation of single muscle contraction? APartial tetanus * BHolotetanus CA series of single contractions D Muscle contructure EAsynchronous tetanus 107 A patient takes cholagogues. What other process besides biliary excretion will be stimulated? AIntestinal motility * BGastric juice secretion CPancreatic juice secretion D Gastric motor activity EWater absorption 108 Packed cell volume of a man was 40% before the trauma. What packed cell volume will be observed 24 hours after blood loss of 750 ml? A30% B40% C55% D 45%
  • 25. E50% 109 A patient staggers and walks astraddle. He has hypomyotonia of arm and leg muscles, staccato speech. In what brain section is this affection localized? ACerebellum * BPutamen CCaudate nucleus D Motor cortex ERed nucleus 110 A pregnant woman had her blood group identified. Reaction of erythrocyte agglutination with standard serums of 0αβ (I), Bα (III) groups didn't proceed with standard serum of Aβ (II) group. The blood group under examination is: AАβ (II) * B0αβ (I) CВα (III) D АВ (IV) E- 111 Blood group of a 30 year old man was specified before an operation. His blood is Rh-positive. Reaction of erythrocyte agglutination was absent with standard sera of 0αβ (I), Аβ (II), Вα (III) groups. The blood under examination is of the following group: A0αβ (I) * BАβ (II) CВα (III) D АВ (IV) E- 112 During an experiment the myotatic reflex has been studied in frogs. After extension in a skeletal muscle its reflectory contraction was absent. The reason for it might be a dysfunction of the following receptors: AMuscle spindles * BNociceptors CArticular D Golgi tendon organs ETactile
  • 26. 113 Vagus branches that innervate heart are being stimulated in course of an experiment. As a result of it the excitement conduction from atria to the ventricles was brought to a stop. It is caused by electrophysical changes in the following structures: AAtrioventricular node * BHis' bundle CSinoatrial node D Ventricles EAtria 114 If a man has an attack of bronchiospasm it is necessary to reduce the effect of vagus on smooth muscles of bronchi. What membrane cytoreceptors should be blocked for this purpose? AM-cholinoreceptors * BN-cholinoreceptors Cα-adrenoreceptors D β-adrenoreceptors Eα- and β-adrenoreceptors 115 When water affects mucous membrane of the inferior nasal meatuses, this causes "diver reflex" that provokes: AReflectory apnea * BReflectrory dyspnea CReflectory hyperpnea D Cough EBronchospasm 116 A man weighs 80 kg, after long physical activity his circulating blood volume is reduced down to 5,4 l, hematocrit makes up 50%, whole blood protein is 80 g/l. These blood characteristics are determined first of all by: AWater loss with sweat * BIncreased number of erythrocytes CIncreased protein concentration in plasm D Increased circulating blood volume EIncreased diuresis 117 A 16 year old boy after an illness has diminished function of protein synthesis in liver as a result of vitamin $K$ deficiency. It will cause disturbance of: ABlood coagulation *
  • 27. BErythrocyte sedimentation rate CAnticoagulant generation D Erythropoietin secretion EOsmotic blood pressure 118 In response to a change in body position from horizontal to vertical blood circulation system develops reflectory pressor reaction. Which of the following is its compulsory component? ASystemic constriction of the venous vessels * BSystemic dilatation of the arterial resistive vessels CDecrease in the circulating blood volume D Increase in the heart rate EWeakening of the pumbing ability of heart 119 Examination of a pregnant woman revealed twice as much concentration of fibrinogen in blood plasm. What ESR can this woman have? A40-50 mm/h * B10-15 mm/h C2-12 mm/h D 5-10 mm/h E0-5 mm/h 120 Introduction of a big dose of histamine to an experimental animal caused abrupt drop of arterial pressure as a result of: ADilatation of resistance vessels * BConstriction of resistance vessels CIncrease of heart rate D Decrease of heart rate EDecrease of heart rate and force 121 Systemic arterial pressure of an adult dropped from 120/70 to 90/50 mm Hg that led to reflectory vasoconstriction. The vasoconstriction will be maximal in the following organ: ABowels * BHeart CBrain D Kidneys EAdrenals
  • 28. 122 Microelectrode technique allowed to register a potential following "all-or-none" law and being able of undecremental spreading. Specify this potential: AAction potential * BExcitatory postsynaptic potential CRest potential D Inhibitory postsynaptic potential EReceptor potential 123 Vagus branches that innervate heart are being stimulated during an experiment. This caused reduction of heart rate due to the intensification of the following process (through the cell membrane of cardiac pacemaker): APotassium ion yield * BPotassium ion entry CCalcium ion entry D Calcium ion yield ECalcium and potassium ion yield 124 Rest potential of a cell equals -80 mV. At what stage of action potential did the membrane potential equal +30 mV? AReverse polarization * BAfter hyperpolarization CAfter depolarization D Depolarization E- 125 A 35 year old man got an injury that caused complete disruption of spinal cord at the level of the first cervical segment. What respiration changes will be observed? AIt will come to a standstill * BNo changes will be observed CDiaphragmal respiration will be maintained, thoracic respiration will disappear D Thoracic respiration will be maintained, diaphragmal respiration will disappear EIt will become infrequent and deep 126 A doctor asked a patient to breath out fully after taking a normal breath. What muscles contract during such exhalation? AAbdominal muscles * BExternal intercostal muscles
  • 29. CDiaphragm D Trapezius muscles EPectoral muscles 127 A man was intoxicated with mushrooms. They contain muscarine that stimulates muscarinic cholinoreceptors. What symptoms signalize intoxication with inedible mushrooms? AMyotic pupils * BMydriatic pupils CBronchi dilatation D Increased heart rate ERise of arterial pressure 128 A man presents with increased heart rate, mydriatic pupils, dry mouth. This condition results from the activation of the following system of function regulation: ASympathetic * BParasympathetic CMetasympathetic D Vago-insular EHypothalamo-pituitary-adrenal 129 In course of an experiment a peripheral section of vagus of an expiremental animal is being stimulated. What changes will be observed? AHeart rate fall * BHeart hurry CPupil dilation D Increase of respiration rate EBronchi dilation 130 Voluntary breath-holding caused increase of respiration depth and frequency. The main factor stimulating these changes of external respiration is: AIncreased tension of CO 2 in blood * BIncreased tension of O2 in blood CDecreased tension of O2 in blood D Decreased tension of CO2 in blood EDecreased concentration of H + in blood 131
  • 30. A patient has delayed conduction of excitement through the atrioventricular node. What changes of ECG will be observed? AProlongation of P-Q interval * BProlongation of Q-S interval CNegative T wave D S-T-segment displacement EProlongation of Q-T interval 132 Surface with an intact toad on it was inclined to the right. Tone of extensor muscles became reflectory higher due to the activation of the following receptors: AVestibuloreceptors of utricle and saccule * BVestibuloreceptors of semicircular ducts CMechanoreceptors of foot skin D Photoreceptors of retina EProprioreceptors 133 In course of an experiment a toad's right labyrinth was destroyed. It will cause amyotonia of the following muscles: ARight extensors * BLeft flexors CLeft extensors D Right flexors ERight and left extensors 134 An animal has an increased tonus of extensor muscles. This the result of intensified information transmission to the motoneurons of the spinal cord through the following descending pathways: AVestibulospinal * BMedial corticospinal CReticulospinal D Rubrospinal ELateral corticospinal 135 Workers of a hothouse farm work under conditions of unfavourable microclimate: air temperature is +37oC, relative humidity is 90%, air speed is 0,2 m/s. The way of heat emission under these conditions will be: AEvaporation * BHeat conduction CConvection
  • 31. D Radiation EAll the ways 136 Lungs of a preterm infant have areas of atelectasis (pulmonary collapse). The main cause is: ASurfactant deficiency * BIncreased viscous resistance CUnderdeveloped inspiration muscles D Diminished force of surface tension of lungs ESurfactant excess 137 Vagi of an experimental animal were cut on both sides. What respiration changes will be observed? AIt will become deep and infrequent * BIt will become shallow and frequent CIt will become deep and frequent D It will become shallow and infrequent ENo changes will be observed 138 A cardiac electric stimulator was implanted to a 75 year old man with heart rate of 40 bpm. Thereafter the heart rate rose up to 70 bpm. The electric stimulator has undertaken the function of the following heart part: ASinoatrial node * BAtrioventricular node CHis' bundle branches D His' bundle fibers EPurkinje's fibers 139 A patient came to the hospital complaining about quick fatigability and apparent muscle weakness. Examination revealed an autoimmune disease that causes disorder of functional receptor condition in neuromuscular synapses. What transmitter will be blocked? AAcetylcholine * BNoradrenalin CDopamine D Serotonin EGlycine 140
  • 32. Which muscle contraction will be observed in the upper extremity during holding (not moving) a load in a certain position? AIsometric * BIsotonic CAuxotonic D Concentric EExcentric 141 Examination of a 35 year old patient revealed high acidity of gastric juice. What receptors should be blocked in order to reduce it? AHistamine * Bα 1-adrenoreceptors Cα2-adrenoreceptors D β1-adrenoreceptors Eβ2-adrenoreceptors 142 A young woman who entered a production department where it strongly smelt of paints and varnishes had a bronchospasm. This reflex was caused by irritation of the following receptors: AIrritant * BJuxtaglomerular CPleura receptors D Central chemoreceptors EPeripheral chemoreceptors 143 A 60-year-old patient presents with weakened peristaltic activity of the bowels. Which of the following foodstuffs would stimulate peristalsis most of all? ABrown bread * BWhite bread CMeat D Lard ETea 144 An isolated muscle fiber is under examination. It was established that the threshold of stimulation force became significantly lower. What is the cause of this phenomenon? AActivation of sodium channels of membrane * BActivation of potassium channels of membrane CInactivation of sodium channels of membrane D Inactivation of potassium channels of membrane
  • 33. EBlock of energy production in the cell 145 It was established that agglutination of the recipient’s blood erythrocytes had been caused by the standard sera from the I and II groups. Serum from the III group as well as anti-Rh serum hadn‘t provoke any agglutination. Which blood group and rhesus is allowed to be transfused this recipient? AB, α (III) Rh - BA, β (II) Rh- C0, α,β, (I) Rh+ D AB (IV), Rh+ EAB (IV), Rh- 146 A patient consumed a lot of reach in proteins food that caused increase of rate of proteolytic enzymes of pancreatic juice. It is also accompanied by increase of rate of the following enzyme: ATripsin * BPepsin CEnterokinase D Gastricsin ERenin 147 In course of an experiment thalamocortical tracts of an animal were cut. What type of sensory perception remained intact? AOlfactory * BAuditory CExteroreceptive D Visual ENociceptive 148 Due to the use of poor-quality measles vaccine for preventive vaccination, a 1-year-old child has developed an autoimmune renal injury. The urine was found to contain macromolecular proteins. What process of urine formation has been disturbed? AFiltration * BReabsorption CSecretion D Reabsorption and secretion ESecretion and filtration
  • 34. 149 In course of an experiment there has been an increase in the nerve conduction velocity. This may be caused by an increase in the concentration of the following ions that are present in the solution around the cell: ANa+ BK+ and Cl- CK+ and Na+ D Ca2+ and Cl- ECa2+ 150 A patient has increased thickness of alveolar-capillary membrane caused by a pathologic process. The direct consequence will be reduction of the following value: ADiffusing lung capacity * BOxygen capacity of blood CRespiratory minute volume D Alveolar ventilation of lungs EExpiratory reserve volume